Blackboard EMQs Neuro Flashcards

1
Q

A 30 year old man has been picked up in the street by police. He was initially drowsy but is now agitated & aggressive. His trousers are wet with urine.

For each of the following patients choose the single most likely diagnosis from the list of options.
A.	Delerium tremens
B.	Cerebrovascular accident
C.	Intoxication
D.	Post-ictal state
E.	Hypoxia
F.	Urinary tract infection
G.	Encephalitis
H.	Hypothyroidism
I.	Acute psychosis
J.	Dementia
K.	Hypothermia
L.	Hypoglycaemia
A

C. Intoxication

This man has had a little too much to drink and is simply intoxicated. He has wet himself. There are no signs or symptoms to suggest this is post-ictal. There is nothing to suggest this patient has had a seizure.

How well did you know this?
1
Not at all
2
3
4
5
Perfectly
2
Q

A 75 year old woman has gradually become confused over 3 or 4 years. She forgets the names & birthdays of her family. She gets lost when she goes shopping alone. She sometimes leaves her cooker on all night

For each of the following patients choose the single most likely diagnosis from the list of options.
A.	Delerium tremens
B.	Cerebrovascular accident
C.	Intoxication
D.	Post-ictal state
E.	Hypoxia
F.	Urinary tract infection
G.	Encephalitis
H.	Hypothyroidism
I.	Acute psychosis
J.	Dementia
K.	Hypothermia
L.	Hypoglycaemia
A

J. Dementia

Dementia is a syndrome with cognitive deterioration. Important differentials are depression, normal ageing and mild cognitive impairment, which need to be considered. It is highly prevalent in the elderly age group, affecting 30-50% by the age of 85. This patient has gradually become confused over 3-4 years, so there is a lower cognitive capability than their premorbid ‘baseline’ level and the progressive slow course suggests a degenerative process. A good history is necessary, from the patient, family, carers and friends to find out what the changes in cognitive function are and whether there are any behavioural, language or personality changes. This may also help you to figure out the exact disease process – for example, the patient may give a history of Parkinson’s, those with vascular dementia may have transient neurological symptoms and patients with NPH may have gait disturbances with urinary incontinence. The MMSE is the most widely used screening test for cognitive function and a score <24 is widely accepted as abnormal. Lab tests cannot diagnose dementia, but are done to find any potentially reversible causes or contributers such as hypothyroidism. A head CT or MRI is also indicated just in case there is a reversible cause like a tumour or hydrocephalus, and to aid diagnosis of the cause of dementia.

How well did you know this?
1
Not at all
2
3
4
5
Perfectly
3
Q

A 20 year old man is irritable & confused. He appears disturbed by loud noises. He is also complaining of a headache & has a pyrexia & mild neck stiffness.

For each of the following patients choose the single most likely diagnosis from the list of options.
A.	Delerium tremens
B.	Cerebrovascular accident
C.	Intoxication
D.	Post-ictal state
E.	Hypoxia
F.	Urinary tract infection
G.	Encephalitis
H.	Hypothyroidism
I.	Acute psychosis
J.	Dementia
K.	Hypothermia
L.	Hypoglycaemia
A

G. Encephalitis

Encephalitis (brain parenchyma inflammation) is not to be confused with meningitis where the meninges is inflammed (although meningoencephalitis can exist). In encephalitis, there is an altered state of consciousness (this patient is confused) and may there also be focal neurology. There is also a fever (exceptions, however, are subacute sclerosing panencephalitis, VZV and HCV). Other typical features include headache and seizures. It is a medical emergency and people are the extremes of age are more at risk. The list of possible causes is massive. If you suspect this diagnosis, empiral IV acyclovir should be started before the results of any investigations are known. A large proportion are caused by HSV and empirical therapy is backed up by RCTs which show improved mortality. You can change the treatment if the cause is known, for instance, ganciclovir for CMV. You should look for a rash which may give a clue to the aetiology. For example, vesicular patterns are seen in VZV, HSV and enteroviruses, EBV causes a maculopapular pattern after ampicillin, Lyme disease pathognomically gives erythema migrans whereas erythema nodosum might make you think of TB. The patient may also have animal/insect bites and a careful and thorough history is important.

How well did you know this?
1
Not at all
2
3
4
5
Perfectly
4
Q

A 75 year old woman was found on the floor at home having collapsed. She is drowsy & confused & has been incontinent of urine. She is shaking. Her pulse is 50 beats per min & her ECG shows J waves

For each of the following patients choose the single most likely diagnosis from the list of options.
A.	Delerium tremens
B.	Cerebrovascular accident
C.	Intoxication
D.	Post-ictal state
E.	Hypoxia
F.	Urinary tract infection
G.	Encephalitis
H.	Hypothyroidism
I.	Acute psychosis
J.	Dementia
K.	Hypothermia
L.	Hypoglycaemia
A

K. Hypothermia

Body temperature <35 degrees Celsius is the defining feature of hypothermia. ECG changes may appear as core temperature hits 32-33, with acute ST elevation and J waves (also called Osborn waves), which are pathognomic. This is an extra deflection at the end of the QRS and I suggest you go look at some images so it sticks. Rectal temperature is the best measure of core body temperature (followed by ear, then mouth). This patient needs to be removed from the cold environment, any wet or cold clothing removed, and then warmed with blankets. The patient will need constant monitoring and warm IV fluids, and the airway needs to be secured. Other signs this patient has include bradycardia, confusion (suggesting very severe hypothermia) and the urinary incontinence may be linked with diuresis induced by the cold, where the kidney effectively loses its concentrating ability. Those at risk of hypothermia include alcoholics and druggies (affects judgement so they fail to respond to the cold and alcohol, as well all know, causes a subjective feeling of warmth when you’re actually cold). Those with altered mental function like dementia, the homeless, elderly and infants/young children are all at increased risk

How well did you know this?
1
Not at all
2
3
4
5
Perfectly
5
Q

A 20 year old man, who was picked up by the police, is behaving irrationally & is confused & irritable. He is pale & sweaty & smells of alcohol. He keeps asking for biscuits.

For each of the following patients choose the single most likely diagnosis from the list of options.
A.	Delerium tremens
B.	Cerebrovascular accident
C.	Intoxication
D.	Post-ictal state
E.	Hypoxia
F.	Urinary tract infection
G.	Encephalitis
H.	Hypothyroidism
I.	Acute psychosis
J.	Dementia
K.	Hypothermia
L.	Hypoglycaemia
A

L. Hypoglycaemia

This patient is hypoglycaemic and has associated signs and symptoms (both sympathoadrenal and neuroglycopenic): he is confused, sweaty, pale, irritable and clearly hungry as he is asking for biscuits. He also smells of alcohol. Alcohol consumption decreases hepatic production of glucose and will contribute to hypoglycaemia. This patient needs glucose and/or glucagon before considering why he has become so hypoglycaemic in the first place. Can you think of some reasons?

How well did you know this?
1
Not at all
2
3
4
5
Perfectly
6
Q

A 30 year old man had an appendicectomy 2 days ago & is now agitated & confused. He is sweaty & has a marked tremor of his hands. He claims that his sleep was disturbed by insects in his bed.

For each of the following patients choose the single most likely diagnosis from the list of options.
A.	Delerium tremens
B.	Cerebrovascular accident
C.	Intoxication
D.	Post-ictal state
E.	Hypoxia
F.	Urinary tract infection
G.	Encephalitis
H.	Hypothyroidism
I.	Acute psychosis
J.	Dementia
K.	Hypothermia
L.	Hypoglycaemia
A

A. Delerium tremens

The Medical Council on Alcohol has a publication called ‘Alcohol and Health’ which is aimed at medical students and newly qualified doctors. I would recommend giving it a read. DT occurs as a result of alcohol withdrawal, although uncommonly (<5%) and usually starts 48-72 hours after cessation. It is characterised by a coarse tremor, agitation, fever, tachycardia, confusion, delusions and hallucinations. This patient also describes formication, which is a symptom that can be seen in alcohol withdrawal.

How well did you know this?
1
Not at all
2
3
4
5
Perfectly
7
Q

A 66 year old woman presents with fatigue, breathlessness & paraesthesiae in all limbs. Examination reveals pallor, loss of position sense & impaired vibration sense.

For each patient, choose the most likely diagnosis from the list of options.
A.	Drug induced
B.	Porphyria
C.	Diabetes Mellitus
D.	Sarcoid
E.	Trauma
F.	Amyloid
G.	Paraneoplastic syndrome
H.	Alcohol
I.	Renal failure
J.	Vitamin B12 deficiency
A

J. Vitamin B12 deficiency

There is loss of proprioception and vibration indicating degeneration of the dorsal column-medial lemniscus pathway. The patient may also have a positive Romberg as a result. Parasthesias are also common in B12 deficiency indicating peripheral neuropathy. Pallor and signs of frank anaemia are late signs. It is important to remember that hepatic B12 stores last for many years so B12 deficiency depends on chronic deficiency over a long period of time. In general, deficiency is caused by decreased intake, decreased gastric breakdown from food or malabsorption from the GIT. Serum B12 is a useful investigation to initially conduct and will guide further investigations and treatment. You should consider an underlying malabsorptive process such as coeliac or Crohn’s as a possible cause.

How well did you know this?
1
Not at all
2
3
4
5
Perfectly
8
Q

A 40 year old man with pulmonary tuberculosis is in the second month of treatment with isoniazid, rifampicin & pyrazinamide. He complains of a burning sensation in his hands & feet. There is impaired sensation to pin prick & light touch.

For each patient, choose the most likely diagnosis from the list of options.
A.	Drug induced
B.	Porphyria
C.	Diabetes Mellitus
D.	Sarcoid
E.	Trauma
F.	Amyloid
G.	Paraneoplastic syndrome
H.	Alcohol
I.	Renal failure
J.	Vitamin B12 deficiency
A

A. Drug induced

Pulmonary TB treatment involves 2 months of an intial phase of treatment with 4 months of continuation with just isoniazid and rifampicin. Isoniazid acts only on mycobacteria and inhibits mycolic acid synthesis. Isoniazid can commonly cause peripheral neuropathy (which is why pyridoxine (vitamin B6) 10mg daily is given as prophylaxis, as it is because of deficiency of this substance which leads to CNS and PNS effects). The peripheral neuropathy is more likely to occur if the patient has co-existing risk factors like DM, CRF, pregnancy and HIV, or is malnourished.

How well did you know this?
1
Not at all
2
3
4
5
Perfectly
9
Q

An 18 year old woman is admitted with very severe abdominal pain & confusion. She is noted to have bilateral wrist drop. She had recently started the oral contraceptive pill.

For each patient, choose the most likely diagnosis from the list of options.
A.	Drug induced
B.	Porphyria
C.	Diabetes Mellitus
D.	Sarcoid
E.	Trauma
F.	Amyloid
G.	Paraneoplastic syndrome
H.	Alcohol
I.	Renal failure
J.	Vitamin B12 deficiency
A

B. Porphyria

There are many types of porphyria. This patient has acute intermittent porphyria, which is characterised by symptoms like the ones this patient describes – abdominal pain, peripheral motor neuropathy, mental symptoms like confusion. These symptoms, certainly in EMQs, can be trigged by the use of certain drugs which are known to provoke AIP attacks. In reality, the list of drugs is pretty vast and include most CYP450 inducers, but in EMQs, alcohol and the OCP are common. Alcohol also induces an enzyme called delta-aminolevulinic acid synthase, which can exacerbate AIP. It is probably not worth learning the pathways unless you plan on sitting USMLE. AIP is a genetic disorder where there is a partial deficiency of PBGD (the third enzyme in the haem biosynthetic pathway). Treatment of acute attacks involves IV haem arginate with adjunctive dextrose IV. The pain is thought to be neuropathic in origin. The patient may complain of red/browny urine due to increased urinary excretion of intermediates in the haem pathway.

How well did you know this?
1
Not at all
2
3
4
5
Perfectly
10
Q

A 67 year old overweight Asian woman presents with painful feet. Direct questioning revealed that she has had nocturia for the last 3 months.

For each patient, choose the most likely diagnosis from the list of options.
A.	Drug induced
B.	Porphyria
C.	Diabetes Mellitus
D.	Sarcoid
E.	Trauma
F.	Amyloid
G.	Paraneoplastic syndrome
H.	Alcohol
I.	Renal failure
J.	Vitamin B12 deficiency
A

C. Diabetes mellitus

Nocturia with diabetic peripheral neuropathy in an overweight woman (obesity and insulin resistance are linked) point to DM. Symptomatic patients need a single random blood glucose of >11.1 or single fasting glucose of >7. Asymptomatic patients need two separate elevated readings for a diagnosis. Alternatively if there are borderline results, an OGTT can be conducted to see if plasma glucose is raised >11.1 two hours after an oral glucose load of 75g. A patient is said to have impaired fasting glucose if fasting glucose falls between 6.1-6.9. Impaired glucose tolerance is present if plasma glucose 2 hours after oral glucose load in OGTT falls between 7.8-11.0. First line intervention in this situation, and in newly diagnosed DM is diet and lifestyle advice and changes.

How well did you know this?
1
Not at all
2
3
4
5
Perfectly
11
Q

A 55 year old smoker presents with rapidly progressing weight loss with severe burning pain in his hands & feet. Chest x-ray shows a small round opacity in the right upper lobe.

For each patient, choose the most likely diagnosis from the list of options.
A.	Drug induced
B.	Porphyria
C.	Diabetes Mellitus
D.	Sarcoid
E.	Trauma
F.	Amyloid
G.	Paraneoplastic syndrome
H.	Alcohol
I.	Renal failure
J.	Vitamin B12 deficiency
A

G. Paraneoplastic syndrome

Paraneoplastic syndrome, is something which occurs as a result of cancer (but not due to the local cancer cells itself). This patient has lung cancer and paraneoplastic manifestations of sensory neuropathy associated with small cell lung cancer. Small cell lung cancer is treated with chemotherapy and is also associated with SIADH and ectopic ACTH.

How well did you know this?
1
Not at all
2
3
4
5
Perfectly
12
Q

A 30 year old woman has developed ear pain & facial weakness. On otoscopy she has an inflamed, bulging tympanic membrane.

For each patient choose the most likely underlying diagnosis from the list of options.
A.	Stroke
B.	Trauma
C.	Sarcoidosis
D.	Ramsay Hunt syndrome
E.	Otitis media
F.	Multiple sclerosis
G.	Bell’s palsy
H.	Post-meningitis
I.	Brainstem tumour
J.	Parotid tumours
K.	Cholesteatoma
A

E. Otitis media

Otitis media is infection of the middle ear and can occur as a result of complicated respiratory illness. The otalgia this patient describes is characteristic, and there may be decreased hearing, vomiting and fever, usually in the presence of a viral respiratory infection. Examination with an otoscope is diagnostic and will reveal an inflammed bulging tympanic membrane with decreased mobility. The membrane may be pink, red, yellow or white. Treatment involves analgesia, and may also include antibiotics. Complications can include the facial weakness this patient has developed in the form of a CNVII palsy, and perforation of the eardrum, mastoiditis and sigmoid sinus thrombosis.

How well did you know this?
1
Not at all
2
3
4
5
Perfectly
13
Q

A 35 year old woman has suddenly developed facial palsy. 6 months before this, she had an episode of blurred vision & unsteadiness. On examination, she has mild ataxia and an afferent pupillary defect.

For each patient choose the most likely underlying diagnosis from the list of options.
A.	Stroke
B.	Trauma
C.	Sarcoidosis
D.	Ramsay Hunt syndrome
E.	Otitis media
F.	Multiple sclerosis
G.	Bell’s palsy
H.	Post-meningitis
I.	Brainstem tumour
J.	Parotid tumours
K.	Cholesteatoma
A

F. Multiple sclerosis

MS is a demyelinating CNS condition which is characterised by 2 or more episodes of neurological dysfunction which are separated in both time and space. This person has had blurred vision and now 6 months later has developed facial palsy. A RAPD is seen in a Marcus Gunn pupil which is tested for with the swinging light test in a CN examination and can be caused by any disease affecting the optic nerve. MS classically presents in white women aged 20-40 with temporary visual/sensory loss although any presentation can occur. MRI is a sensitive test but less specific than spinal MRI, however, spinal MRI is abnormal in fewer cases. Treatment aims at treating the attack, preventing future attacks and symptomatic treatment of problems like bladder dysfunction, pain and fatigue.

How well did you know this?
1
Not at all
2
3
4
5
Perfectly
14
Q

A 70 year old man has suddenly developed facial weakness, which was preceded by 2 days of severe left ear pain, vertigo & deafness. On examination, he has red vesicles in his ear canal and on the hard palate.

For each patient choose the most likely underlying diagnosis from the list of options.
A.	Stroke
B.	Trauma
C.	Sarcoidosis
D.	Ramsay Hunt syndrome
E.	Otitis media
F.	Multiple sclerosis
G.	Bell’s palsy
H.	Post-meningitis
I.	Brainstem tumour
J.	Parotid tumours
K.	Cholesteatoma
A

D. Ramsay Hunt syndrome

Ramsay Hunt syndrome is reactivation of VZV in the geniculate ganglion and the syndrome consists of CNVII palsy caused by herpes zoster. There is ear pain and an erythematous vesicular rash in the ear canal and on the hard palate which is characteristic of VZV. There is also vertigo in this patient and hearing loss, which is due to CNVIII being affected, due to being in close proximity to the geniculate ganglion. Acyclovir is needed but the damage may well be permanent.

How well did you know this?
1
Not at all
2
3
4
5
Perfectly
15
Q

A 50 year old woman has developed complete palsy of the left side of the face including the forehead. She also has mild facial pain & watering of the eye on that side. Her sense of taste is impaired.

For each patient choose the most likely underlying diagnosis from the list of options.
A.	Stroke
B.	Trauma
C.	Sarcoidosis
D.	Ramsay Hunt syndrome
E.	Otitis media
F.	Multiple sclerosis
G.	Bell’s palsy
H.	Post-meningitis
I.	Brainstem tumour
J.	Parotid tumours
K.	Cholesteatoma
A

G. Bell’s palsy

Bell’s palsy is idiopathic unilateral LMN CNVII palsy. It is a diagnosis of exclusion as the aetiology is unknown, so the CNVII palsy of RHS in the previous question is not Bell’s palsy. Corticosteroids are effective and surgical decompression can be considered in severe cases. Eye protection should be considered too. This condition tends to resolve itself by 4-6 months. Think about what you’d expect to find on examination of CNVII. CNVII supplies taste to the anterior 2/3 of the tongue. Think back to anatomy. CNVII supplies all the muscles of facial expression, stapedius and some other muscles (posterior belly of digastric, stylohyoid and occipitofrontalis). It is sensory to the external auditory meatus (via the nervus intermedius). It is parasympathetic via the same nerve, to supply the submandibular and sublingual glands. The same nerve also carries taste sensation in the anterior two thirds of the tongue, and innervates the palate. Do you still remember the branches of the facial nerve?

How well did you know this?
1
Not at all
2
3
4
5
Perfectly
16
Q
A 56 year old woman with a history of atrial fibrillation develops sudden weakness of the right side of her face. She is still able to wrinkle both sides of her forehead &amp; her smile is symmetrical.
For each patient choose the most likely underlying diagnosis from the list of options.
A.	Stroke
B.	Trauma
C.	Sarcoidosis
D.	Ramsay Hunt syndrome
E.	Otitis media
F.	Multiple sclerosis
G.	Bell’s palsy
H.	Post-meningitis
I.	Brainstem tumour
J.	Parotid tumours
K.	Cholesteatoma
A

A. Stroke

This is an UMN CNVII lesion as the forehead is spared. A stroke is a cause of an UMN lesion (as is a tumour, although both of these in the brainsteam can cause a LMN lesion) and the patient’s history of AF (which can throw off an emboli to cause an ischaemic stroke) and sudden onset of symptoms is highly suggestive. It is important is perform a CT head exclude a haemorrhagic aetiology and consider thrombolysis with tPA if within the 4.5 hour window and there are no contraindications. Thrombolysis is done with alteplase at 10% bolus, 90% infusion at a dose of 0.9 mg/kg. Presentation after the 4.5 hour window is managed with aspirin. The Bamford/Oxford Stroke Classification subtypes ischaemic stroke according to vascular territory of infarction. After initial management, stroke care involves the ethos of an MDT environment with rehabilitation.

How well did you know this?
1
Not at all
2
3
4
5
Perfectly
17
Q

A 35 year old man has developed a slowly progressive right-sided facial palsy with deafness & tinnitus. As well as facial asymmetry, he is unable to abduct his right eye. His father had been similarly affected.

For each patient choose the most likely underlying diagnosis from the list of options.
A.	Stroke
B.	Trauma
C.	Sarcoidosis
D.	Ramsay Hunt syndrome
E.	Otitis media
F.	Multiple sclerosis
G.	Bell’s palsy
H.	Post-meningitis
I.	Brainstem tumour
J.	Parotid tumours
K.	Cholesteatoma
A

I. Brainstem tumour

Let us review the nerves that are damaged here – CNVII, CNVIII, CNVI. A cholesteatoma can involve VII (rarely) and tinnitis and hearing loss tends to be what the patient presents with but this will not explain LR dysfunction. Also, there will likely be a purulent discharge from the ear which is malodorous, and examination by a hopefully not incompetent doctor will include otoscopy, where typically, you would see crust in the attic, pars flaccida or tensa and possible perforation of the ear drum. The only thing on the list which would explain all three nerve lesions is a brainstem tumour, which also agrees with the slowly progressive presentation (the tumour is likely growing). The question you might be asking is, where or what is this lesion? Have a think about it and I will tell you what I think it is below. I think this is a lesion at the cerebellopontine angle – a vestibular schwannoma (acoustic neuroma). There is likely to be facial numbness in this patient too i.e. CNV, VII, VIII lesions. And how do you explain the CNVI lesion? Raised intracranial pressure. It helps if you know your anatomy so you can try and figure out the cause of CN lesions. For example, nerves III, IV, Va and VI makes you think of what?

How well did you know this?
1
Not at all
2
3
4
5
Perfectly
18
Q

An elderly lady is found collapsed & confused at home by her district nurse. She had been fit & well until 3 days ago when she started behaving oddly & yesterday her neighbour found her wandering around in the road in her dressing gown. On examination, she has a low-grade fever, a tender abdomen & an unpleasant smell.

For each patient choose the most appropriate diagnosis from the list.
A.	Diabetic ketoacidosis
B.	Alcohol withdrawal
C.	Phenytoin toxicity
D.	Schizophrenia
E.	Subdural haematoma
F.	Ecstasy overdose
G.	Meningitis
H.	Urinary tract infection
I.	Dementia
J.	Hepatic encephalopathy
A

H. Urinary tract infection

10% of women >70 have a UTI. This is the cause of this elderly lady’s confusion. Her set of symptoms (confusion, fever, abdominal tenderness) are not explained by any of the other conditions given on the list. A UTI, in uncomplicated cases is most commonly caused by E coli. It is diagnosed with a dipstick and urine MC+S from an MSU sample. Have a think about what the dipstick would show. Antibiotic therapy should be guided by local sensitivities and guidelines, or MC+S results. Nitrofurantoin or co-trimoxazole could be used.

How well did you know this?
1
Not at all
2
3
4
5
Perfectly
19
Q

A 62 year old hotel owner is in hospital for investigation of his jaundice. When you come to see him in the morning he is slurring his speech & doesn’t seem to know where he is. You can’t help noticing that his abdomen is grossly distended & his breath smells strangely sweet.

For each patient choose the most appropriate diagnosis from the list.
A.	Diabetic ketoacidosis
B.	Alcohol withdrawal
C.	Phenytoin toxicity
D.	Schizophrenia
E.	Subdural haematoma
F.	Ecstasy overdose
G.	Meningitis
H.	Urinary tract infection
I.	Dementia
J.	Hepatic encephalopathy
A

J. Hepatic encephalopathy

This patient has decompensated chronic liver disease (he is in liver failure) which has resulted in neurological symptoms associated with hepatic encephalopathy. The brain is exposed to ammonia which bypasses the liver by portosystemic shunting. It is a diagnosis of exclusion and tests will need to be conducted to rule out other potential causes of confusion. The findings of jaundice, ascites and fetor hepaticus (liver failure) are all signs of liver disease. Think about the other signs you might see like spider naevi and palmar erythema. This patient may also have asterixis which is a coarse flapping tremor. HE is likely caused by a host of factors. This patient’s LFTs will be abnormal and he is likely to have coagulopathy too (PT will be elevated).

How well did you know this?
1
Not at all
2
3
4
5
Perfectly
20
Q

A 21 year old diabetic student is brought into A&E by his friends. They say he has been acting weirdly all night & wondered if he was on drugs. A couple of hours ago he developed a fever & started vomiting. When you meet him he seems very irritable & is complaining that the lights are too bright. Examination reveals an erythematous rash over his back

For each patient choose the most appropriate diagnosis from the list.
A.	Diabetic ketoacidosis
B.	Alcohol withdrawal
C.	Phenytoin toxicity
D.	Schizophrenia
E.	Subdural haematoma
F.	Ecstasy overdose
G.	Meningitis
H.	Urinary tract infection
I.	Dementia
J.	Hepatic encephalopathy
A

G. Meningitis

This patient has meningitis. Universities are common sites of outbreaks due to crowding. Commonly there will be a headache, fever and nuchal rigidity. There may also be an altered mental status, confusion, photophobia and vomiting. Kernig’s sign is uncommon but is positive when attempts to extend the leg are met with resistance when the patient is supine with the thigh flexed to 90 degrees. Another uncommon sign is Brudzinski’s sign and a petechial/purpuric rash, typically associated with meningococcal meningitis. CT head should be considered before LP if there is any evidence of raised ICP. An LP will confirm the diagnosis with bacterial meningitis showing a low CSF glucose, elevated CSF protein and positive CSF culture/gram stain or meningococcal antigen.

How well did you know this?
1
Not at all
2
3
4
5
Perfectly
21
Q

You are called to see a 45 year old man who is known to have suffered a subarachnoid haemorrhage 2 months ago. In hospital he was prescribed an anti-convulsant to reduce the risk of seizure & has continued to take it, despite being discharged a fortnight ago. Over the past week he has become increasingly confused, lethargic & ataxic. On examination, he has nystagmus & an intention tremor & shows past-pointing

For each patient choose the most appropriate diagnosis from the list.
A.	Diabetic ketoacidosis
B.	Alcohol withdrawal
C.	Phenytoin toxicity
D.	Schizophrenia
E.	Subdural haematoma
F.	Ecstasy overdose
G.	Meningitis
H.	Urinary tract infection
I.	Dementia
J.	Hepatic encephalopathy
A

C. Phenytoin toxicity

PHT has an unpredictable pharmacokinetic behaviour. It is 80-90% bound to albumin (competitive binding by drugs such as salicylates increase free PHT). It is metabolised by hepatic mixed function oxidase and metabolism can be either induced or inhibited by drugs which share the same hepatic enzymes. It has a narrow therapeutic range of around 40-100 micromol/l and there is a lot of individual variation in the plasma concentration achieved with a fixed dose and unwanted effects tend to occur >100 micromol/l. The symptoms include those seen like nystagmus, confusion, headache, ataxia and vertigo. Chronic use is also associated with unwanted effects like gum hyperplasia and use is associated with fetal malformations like cleft palate (associated with epoxide formation in metabolism).

How well did you know this?
1
Not at all
2
3
4
5
Perfectly
22
Q

A 37 year old man has had half his ear bitten off in a fight & is admitted under the plastic surgeons. After being on the ward for a day & a half he becomes extremely agitated, claiming to see spiders & snakes crawling up the walls. Examination shows him to be tachycardic & sweaty but is otherwise unremarkable.

For each patient choose the most appropriate diagnosis from the list.
A.	Diabetic ketoacidosis
B.	Alcohol withdrawal
C.	Phenytoin toxicity
D.	Schizophrenia
E.	Subdural haematoma
F.	Ecstasy overdose
G.	Meningitis
H.	Urinary tract infection
I.	Dementia
J.	Hepatic encephalopathy
A

B. Alcohol withdrawl

This is a case of alcohol withdrawal experienced by some 40% of alcohol abusers who subsequently come off alcohol. Scary auditory and visual hallucinations can occur along with the symptoms described including tachycardia, anxiety, sweating, tremor, nausea, retching and a mild pyrexia. Acutely, this should be treated with a BDZ such as chlordiazepoxide. It can progress to delirium tremens with seizures, hallucinations, coma and death.

How well did you know this?
1
Not at all
2
3
4
5
Perfectly
23
Q

A 90 year old woman who uses a Zimmer frame because of her OA & general frailty. She has a 3 day weakness of her left arm, which has worsened. Yesterday, she could not walk & became confused & incontinent. You find a flaccid paralysis of her arm & weakness of power in her leg.

Choose the most likely diagnosis
A.	Food poisoning
B.	Alzheimer’s disease
C.	Electrolyte disturbance
D.	Severe anxiety state
E.	Urinary tract infection
F.	Opiate poisoning
G.	Chronic subdural haematoma
H.	Alcohol withdrawal
I.	Multi-infarct dementia
J.	Digoxin toxicity
A

G. Chronic subdural haematoma

A subdural occurs due to blood collecting between the dura mater and the arachnoid mater surrounding the brain. It may be arterial or venous although is most often venous. The disease course varies, and in this case this is chronic with confusing, incontinence (both bowel and bladder can occur), and focal neurology. There is neurological deficit evident so surgery will be indicated. The cause is trauma and this frail old lady will most likely have had many falls. Advanced age is associated with chronic subdurals. Chronic subdural haematomas are usually hypodense on head CT. It is important in the examination to look for signs of trauma such as scalp abrasions and bruises. This patient is symptomatic and surgical options include twist-drill craniotomy with drainage (a bedside procedure where a hand drill is used to gain access to the subdural space and then a catheter is placed to act as a drain). Standard craniotomy is also an option, as is the creation of a burr hole. Remember that extradural haematomas classically have a ‘lucid interval’ and occur in younger patients, usually with an associated skull fracture, and CT of the haematoma does not cross suture lines.

How well did you know this?
1
Not at all
2
3
4
5
Perfectly
24
Q

A very old patient in a nursing home, who has had 3 strokes & who is catheterised. She takes aspirin, & bendrofluazide for her blood pressure. She has become confused with a fever.

Choose the most likely diagnosis
A.	Food poisoning
B.	Alzheimer’s disease
C.	Electrolyte disturbance
D.	Severe anxiety state
E.	Urinary tract infection
F.	Opiate poisoning
G.	Chronic subdural haematoma
H.	Alcohol withdrawal
I.	Multi-infarct dementia
J.	Digoxin toxicity
A

E. Urinary tract infection

The indwelling catheter is a significant risk factor for UTI, which can present with a fever and confusion. Other risk factors this patient has: advanced age and post-menopause. It is diagnosed with a dipstick and urine MC+S from an MSU sample. Have a think about what the dipstick would show. Antibiotic therapy should be guided by local sensitivities and guidelines, or MC+S results. Nitrofurantoin is usually effective.

How well did you know this?
1
Not at all
2
3
4
5
Perfectly
25
Q

A 30 year old man who lost his job as a publican 3 days ago. He has become confused, sweaty, & his limbs shake. He is very scared by these symptoms. His pulse is 120 & his blood pressure 100/60.

Choose the most likely diagnosis
A.	Food poisoning
B.	Alzheimer’s disease
C.	Electrolyte disturbance
D.	Severe anxiety state
E.	Urinary tract infection
F.	Opiate poisoning
G.	Chronic subdural haematoma
H.	Alcohol withdrawal
I.	Multi-infarct dementia
J.	Digoxin toxicity
A

H. Alcohol withdrawal

This is a case of alcohol withdrawal experienced by some 40% of alcohol abusers (this man is a publican) who subsequently come off alcohol. Symptoms include confusion, tachycardia, anxiety, sweating, tremor, nausea, retching and a mild pyrexia. Acutely, this should be treated with a BDZ such as chlordiazepoxide. It can progress to delirium tremens (which this patient may have) with seizures, hallucinations, coma and death.

How well did you know this?
1
Not at all
2
3
4
5
Perfectly
26
Q

A 70 year old woman was brought in by her husband. She has become more forgetful over the last few months. Last night she had let a pan of water boil dry & almost burnt the house down.

Choose the most likely diagnosis
A.	Food poisoning
B.	Alzheimer’s disease
C.	Electrolyte disturbance
D.	Severe anxiety state
E.	Urinary tract infection
F.	Opiate poisoning
G.	Chronic subdural haematoma
H.	Alcohol withdrawal
I.	Multi-infarct dementia
J.	Digoxin toxicity
A

B. Alzheimer’s disease

This is Alzheimer’s dementia which is a progressive irreversible disorder characterised by memory loss, loss of social function and dimished executive function. The disease runs a deteriorating course and lesions in the brain are characterised by neurofibrillary tangles, plaques of beta amyloid and neurone loss with cortical atrophy. Cholinesterase inhibitors can be used (donepezil, rivastigmine, galantamine). Depression is common in AD and antidepressants may also be indicated. Carer support is crucial and remains the mainstay of treatment with an MDT ethos involving for example, OTs to assess home safety.

How well did you know this?
1
Not at all
2
3
4
5
Perfectly
27
Q

A 19 year old girl found collapsed outside a local nightclub. She is drowsy, confused with abdominal cramps & diarrhoea. She is difficult to examine but you notice bilateral small pupils.

Choose the most likely diagnosis
A.	Food poisoning
B.	Alzheimer’s disease
C.	Electrolyte disturbance
D.	Severe anxiety state
E.	Urinary tract infection
F.	Opiate poisoning
G.	Chronic subdural haematoma
H.	Alcohol withdrawal
I.	Multi-infarct dementia
J.	Digoxin toxicity
A

F. Opiate poisoning

The history in a nightclub suggests opiate OD. Signs include CNS depression, miosis and apnoea. Opiate use is normally associated with constipation but the history and findings of small constricted pupils are still indicative. Naloxone is indicated both therapeutically and diagnostically. If there is a response, then it is diagnostic. Another diagnosis should be sought if the patient is unresponsive. IV is the preferred route of administration although naloxone can be given IM or SC if IV access cannot be established. Ventilatory support is key with 100% oxygen. You can check out Toxbase for a full database on poisons and treatments.

How well did you know this?
1
Not at all
2
3
4
5
Perfectly
28
Q

A 45 year old type 1 diabetic patient presents with a history of feeling light headed. On examination, he is noted to have a blood pressure of 150/90 lying & 125/70 on standing.

Choose the most appropriate diagnosis
A.	Abducent (VI) nerve palsy
B.	Amaurosis fugax
C.	Common peroneal nerve palsy
D.	Sciatica
E.	Peripheral neuropathy
F.	Carpal tunnel syndrome
G.	Ulnar nerve palsy
H.	Occulomotor (III) nerve palsy
I.	Autonomic neuropathy
J.	Retinopathy
K.	Diabetic amyotrophy
A

I. Autonomic neuropathy

Diabetic neuropathy can be autonomic or peripheral. This patient has postural hypotension (measure BP supine and then standing after 1, 2, 3 and sometimes 5 minutes – an abnormal drop when standing is indicative). Other symptoms of autonomic neuropathy include… resting tachycardia (late findings due to vagal impairment), impaired HR variation, erectile dysfunction (affects many diabetic men though is not solely due to autonomic neuropathy), decreased libido, dyspareunia, and urinary symptoms of frequency, urgency, incontinence, nocturia, weak stream and retention. Other symptoms include constipation, faecal incontinence and sweating dysfunction. Fludrocortisone may be helpful.

How well did you know this?
1
Not at all
2
3
4
5
Perfectly
29
Q

A 73 year old, previously fit male presents with difficulty ascending stairs. Abnormalities noted on examination are weakness of knee flexion, which is more pronounced on the left with some wasting of the quadriceps & diminished knee reflexes. He is noted to have glycosuria

Choose the most appropriate diagnosis
A.	Abducent (VI) nerve palsy
B.	Amaurosis fugax
C.	Common peroneal nerve palsy
D.	Sciatica
E.	Peripheral neuropathy
F.	Carpal tunnel syndrome
G.	Ulnar nerve palsy
H.	Occulomotor (III) nerve palsy
I.	Autonomic neuropathy
J.	Retinopathy
K.	Diabetic amyotrophy
A

K. Diabetic amyotrophy

Diabetic amyotrophy, more common in T2DM, is an uncommon peripheral diabetic neuropathic complaint. It presents with severe muscle weakness and pain with proximal thigh muscle atrophy. This patient’s glycosuria suggests undiagnosed T2DM, which in any case, you can guess he has given the question stem is called ‘Diabetic Complications’. The weak knee flexion and quadriceps wasting is typical of diabetic amyotrophy. The reduced reflexes are another sign of peripheral neuropathy.

How well did you know this?
1
Not at all
2
3
4
5
Perfectly
30
Q

A 62 year old male diabetic presents with a sudden onset of double vision. He is noted to have ptosis and a deviation of the right eye down and to the right. The pupils appear equal in size and are reactive to light.

Choose the most appropriate diagnosis
A.	Abducent (VI) nerve palsy
B.	Amaurosis fugax
C.	Common peroneal nerve palsy
D.	Sciatica
E.	Peripheral neuropathy
F.	Carpal tunnel syndrome
G.	Ulnar nerve palsy
H.	Occulomotor (III) nerve palsy
I.	Autonomic neuropathy
J.	Retinopathy
K.	Diabetic amyotrophy
A

H. Occulomotor (III) nerve palsy

A complete (or surgical) third nerve palsy presents with compete ptosis, a dilated pupil and the eye turned down and out and can be caused by a PCA aneurysm. This is an emergency and the patient needs a neurosurgical opinion with MRI/angiography. A partial pupil-sparing (or medical) third nerve palsy presents with partial ptosis without pupillary signs and can be caused by infarction of the nerve i.e. mononeuritis multiplex, due to possible DM or vasculitis like GCA.

Remember from anatomy that CNIII innvervates all the mucles of the eye except SO and LR. CNIII also supplies levator palpebrae superioris and the sphincter pupillae which causes miosis. If the cause is surgical and compressive, the parasympathetic nerve is affected early on, as it lies at the surface of the third nerve. In an infarctive medical third nerve palsy, the centre infarcts, which is affected more than the surface of the nerve. Hence, the pupil is spared and there is partial ptosis.

How well did you know this?
1
Not at all
2
3
4
5
Perfectly
31
Q

A 66 year old male with type 2 diabetes complains of episodes of loss of vision in the right eye that may last up to 2 hours. On examination he is noted to have an irregularly irregular pulse of 70 beats per minute & a blood pressure of 155/95. Fundoscopy is normal.

Choose the most appropriate diagnosis
A.	Abducent (VI) nerve palsy
B.	Amaurosis fugax
C.	Common peroneal nerve palsy
D.	Sciatica
E.	Peripheral neuropathy
F.	Carpal tunnel syndrome
G.	Ulnar nerve palsy
H.	Occulomotor (III) nerve palsy
I.	Autonomic neuropathy
J.	Retinopathy
K.	Diabetic amyotrophy
A

B. Amaurosis fugax

Amaurosis fugax is a transient and painless loss of vision in one eye due to the passage of an embolus into the central retinal artery. The AF has thrown off a cardiac embolus which has passed into the central retinal artery (resulting in a temporary loss of blood flow to the retina and hence loss of vision). This patient needs to be started on aspirin at once while a definitive treatment of the underlying aetiology is sought (treatment of AF). How would you treat AF?

How well did you know this?
1
Not at all
2
3
4
5
Perfectly
32
Q

A 56 year old type 2 diabetic female is admitted with pain in the feet that keeps her awake at night. The only abnormality noted on examination is loss of vibration sensation up to the mid-tibia bilaterally

Choose the most appropriate diagnosis
A.	Abducent (VI) nerve palsy
B.	Amaurosis fugax
C.	Common peroneal nerve palsy
D.	Sciatica
E.	Peripheral neuropathy
F.	Carpal tunnel syndrome
G.	Ulnar nerve palsy
H.	Occulomotor (III) nerve palsy
I.	Autonomic neuropathy
J.	Retinopathy
K.	Diabetic amyotrophy
A

E. Peripheral neuropathy

This is a case of diabetic peripheral sensory neuropathy. This is a microvascular complication of DM and is characterised by peripheral nerve dysfunction. There tends to be loss of sensation typically occuring in a symmetrical ‘glove and stocking’ distribution. Patient’s may also describe a pain (like this one) or unpleasant sensation which is prickling, burning or sticking. Examination should include peripheral pulses, reflexes and sensation to light touch, vibration (128Hz tuning fork), pinprick and proprioception. Any pain can be treated with medications like gabapentin.

How well did you know this?
1
Not at all
2
3
4
5
Perfectly
33
Q

A 25 year old, highly stressed Junior House Officer complains of a headache that has been persistent for weeks. She describes the pain as being “like a tight band around her head”. Over the counter medication has been used to no avail.

Choose the single most likely diagnosis from the list of options.
A.	Congenital heart disease
B.	Tension headache
C.	TIA
D.	Bacterial meningitis
E.	Cervical spondylosis
F.	Stroke
G.	Trigeminal neuralgia
H.	Cerebral tumour
I.	Subarachnoid haemorrhage
J.	Extradural haemorrhage
K.	Encephalitis
L.	Migraine
A

B. Tension headache

A tension headache is commonly triggered by stress and mental tension (also, fatigure and missing meals), hence the name. It is more common in females and those in middle age, and there is a link with lower socioeconomic status, although this does not necessarily represent causation. Symptoms include a dull, non-pulsatile and constricting bilateral pain, which is often described as a band across the patient’s head. It is not severe or disabling but classically worsens as the day progresses. This headache normally responds well to simple analgesics.

How well did you know this?
1
Not at all
2
3
4
5
Perfectly
34
Q

A 40 year old housewife complains of a repeated history of a unilateral throbbing headache lasting several hours for 6 months. The headache is associated with a disturbance of vision. She claims that eating cheese may trigger it.

Choose the single most likely diagnosis from the list of options.
A.	Congenital heart disease
B.	Tension headache
C.	TIA
D.	Bacterial meningitis
E.	Cervical spondylosis
F.	Stroke
G.	Trigeminal neuralgia
H.	Cerebral tumour
I.	Subarachnoid haemorrhage
J.	Extradural haemorrhage
K.	Encephalitis
L.	Migraine
A

L. Migraine

Migraine is a chronic condition, with genetic determinants, which usually presents in early to mid life. The typical migraine aura this patient describes (which can be visual, sensory or speech symptoms) which can occur during or before the headache, is pathognomic, but is not seen in the majority of patients. The aura can be positive phenomena (for example seeing flashing lights) or negative phenomena (for example visual loss). Nausea, photophobia and disability (the headache gets in the way with the patient’s ability to function) accompanying a headache suggest a migraine diagnosis. The headache of a migraine tends to be prolonged if untreated, and tends to be unilateral and pounding (but does not have to be). Tests aim to rule out other differentials, although if the history is compatible and neurological examination is unremarkable, further testing is not needed. Important red flags which make you think of another dangerous cause can be summed up by the mnemonic SNOOP.

Systemic symptoms: fever, weight loss
Neurological symptoms: confusion, impaired consciousness
Onset: sudden or split-second
Older: new-onset/progressive headache, especially in those >50 (making you think of GCA
PMH: first headache, or something different about this one compared to the usual

Treatment of this chronic condition aims at treating acute attacks to restore function. Triptans can be used in specialist care. These are 5HT1 agonists. Effective initial treatment in a primary care setting can involve NSAIDs, which are available OTC. Treatment should be taken as soon as a patient realises they are having an attack and may need to be repeated after the attack. A few patients who have frequent, severe or disabling headaches may require daily prophylaxis such as anticonvulsants, TCAs and beta blockers.

How well did you know this?
1
Not at all
2
3
4
5
Perfectly
35
Q

A 19 year old male 1st year university student complains of a rapidly developing headache & a stiff neck. He has been vomiting & his friends say that he cannot stand to be in bright rooms. Examination reveals a pyrexia of 37.5 degrees Celsius.

Choose the single most likely diagnosis from the list of options.
A.	Congenital heart disease
B.	Tension headache
C.	TIA
D.	Bacterial meningitis
E.	Cervical spondylosis
F.	Stroke
G.	Trigeminal neuralgia
H.	Cerebral tumour
I.	Subarachnoid haemorrhage
J.	Extradural haemorrhage
K.	Encephalitis
L.	Migraine
A

D. Bacterial meningitis

This patient has meningitis. Universities are common sites of outbreaks due to crowding. Commonly there will be a headache, fever and nuchal rigidity. There may also be an altered mental status, confusion, photophobia and vomiting. Kernig’s sign is uncommon but is positive when attempts to extend the leg are met with resistance when the patient is supine with the thigh flexed to 90 degrees. Another uncommon sign is Brudzinski’s sign and a petechial/purpuric rash, typically associated with meningococcal meningitis.

CT head should be considered before LP if there is any evidence of raised ICP. An LP will confirm the diagnosis with bacterial meningitis showing a low CSF glucose, elevated CSF protein and positive CSF culture/gram stain or meningococcal antigen.

How well did you know this?
1
Not at all
2
3
4
5
Perfectly
36
Q

A 70 year old man presents to his GP surgery with repeated episodes left sided hemiparesis. A recent ECG reveals that he is in atrial fibrillation. His symptoms fully resolve within 24 hours.

Choose the single most likely diagnosis from the list of options.
A.	Congenital heart disease
B.	Tension headache
C.	TIA
D.	Bacterial meningitis
E.	Cervical spondylosis
F.	Stroke
G.	Trigeminal neuralgia
H.	Cerebral tumour
I.	Subarachnoid haemorrhage
J.	Extradural haemorrhage
K.	Encephalitis
L.	Migraine
A

C. TIA

A TIA is colloquially called a ‘mini stroke’ with symptoms typically lasting under an hour. An antiplatelet drug such as aspirin is effective secondary prevention if the patient is not already anticoagulated. The patient will be anticoagulated if they have a likely or known cardioembolic source such as AF. Clopidogrel is an alternative in those who do not tolerate aspirin.

How well did you know this?
1
Not at all
2
3
4
5
Perfectly
37
Q

A 55 year old known hypertensive male complains of a sudden devastating occipital headache. He says that he feels as though he had “been kicked in the head” even though he has not experienced any trauma in the last few weeks. He is feeling drowsy & during the examination he loses consciousness.

Choose the single most likely diagnosis from the list of options.
A.	Congenital heart disease
B.	Tension headache
C.	TIA
D.	Bacterial meningitis
E.	Cervical spondylosis
F.	Stroke
G.	Trigeminal neuralgia
H.	Cerebral tumour
I.	Subarachnoid haemorrhage
J.	Extradural haemorrhage
K.	Encephalitis
L.	Migraine
A

I. Subarachnoid haemorrhage

SAH (bleeding into the subarachnoid space) presents with sudden severe headache patients will often describe as the worst headache of their life, and can often be so bad that they feel like they’ve been kicked in the back of the back. Half of all patients lose consciousness and eye pain with exposure to light can also be seen. Altered mental status is common. SAH occurs most commonly in the 50-55 age group and affects women and black people more than men and white people. The most common cause of non-traumatic SAH is an aneurysm which ruptures. Conditions which predispose to aneurysm formation and SAH include adult PKD, Marfan’s, NF1 and Ehlers-Danlos. Cerebral aneurysms arise around the circle of Willis. A CT scan is indicated, and if unrevealing, this should be followed by an LP. Cerebral angiography can confirm the presence of aneurysms. The patient should be stabilised and this followed by surgical clipping or endovascular coil embolisation, the choice is subject to much current controversy sparked by relatively recent research. Complications can commonly occur and include rebleeding, hydrocephalus and vasospasm.

How well did you know this?
1
Not at all
2
3
4
5
Perfectly
38
Q

A 60 year old lady with recent onset unilateral headache made worse by combing her hair. Her thyroid function was in the hypothyroid range & her ESR was 60mm/hr.

Choose the most likely cause of the symptoms from the list of options.
A.	Analgesic rebound headache
B.	Temporal arteritis
C.	Raised intracranial pressure
D.	Concussive syndrome
E.	Subarachnoid haemorrhage
F.	Migraine
G.	Tension headache
H.	Sinusitis
I.	Encephalitis
J.	Meningitis
K.	Brain abscess
L.	Trigeminal neuralgia
M.	Venous sinus thrombosis
N.	Severe hypertension
A

B Temporal arteritis

Temporal arteritis/GCA is a form of vasculitis which commonly affects those >50 and females. The headache typically presents over the temporal/occipital areas and may be accompanied by scalp tenderness – hence made worse by combing her hair. There may additionally be PMR symptoms such as aching and stiffness of the proximal extremities worse after inactivity and with movement. The patient may also have jaw claudication and the artery may be thickened and tender on examination. Systemic symptoms like a low grade fever and fatigue are commonly seen. Treatment is with oral prednisolone which should be started if there is high suspicion of this diagnosis and this should not even be delayed by results of ESR/CRP. IV pulse methylprednisolone is preferred if there are visual or neurological signs/symptoms. The best test to establish a diagnosis is with a temporal artery biopsy, however this should not delay treatment as irreversible blindness is the most common serious consequence of this condition. The arteritis is patchy so an adequate biopsy sample is needed. Histopathology will show a granulomatous inflammatory process, with giant cells present in around half of cases. GCA is unlikely (although 5% occur with normal ESR) if inflammatory markers are normal so an ESR is a quick test and >100 predicts a positive biopsy result (likelihood ratio of 1.9).

How well did you know this?
1
Not at all
2
3
4
5
Perfectly
39
Q

A 30 year old nurse presented with frontal headache after an acute viral illness. The pain was constant, affecting the right side of her head only. On examination there was marked tenderness on the right side of her face, over the maxillary area.

Choose the most likely cause of the symptoms from the list of options.
A.	Analgesic rebound headache
B.	Temporal arteritis
C.	Raised intracranial pressure
D.	Concussive syndrome
E.	Subarachnoid haemorrhage
F.	Migraine
G.	Tension headache
H.	Sinusitis
I.	Encephalitis
J.	Meningitis
K.	Brain abscess
L.	Trigeminal neuralgia
M.	Venous sinus thrombosis
N.	Severe hypertension
A

H. Sinusitis

This patient has acute sinusitis (lasting 4 weeks or less) which is most commonly due to a viral aetiology, like this case. If symptoms last >10 days, this would indicate that the cause is bacterial (mainly Streptococcus pneumoniae and Haemophilus influenzae). This condition is normally self limiting and needs only symptomatic treatment. Antibiotics are not usually recommended unless the patient is immunocompromised or has particularly severe disease. There is facial pain here which can manifest as a headache, and is usually associated with a bacterial cause although can occur with an acute viral cause.

How well did you know this?
1
Not at all
2
3
4
5
Perfectly
40
Q

A 40 year old man presented with generalised pain in his head, which became progressively worse especially in the mornings. Coughing and bending over made his pain significantly worse. He mentioned that he vomited twice on his way to the surgery.

Choose the most likely cause of the symptoms from the list of options.
A.	Analgesic rebound headache
B.	Temporal arteritis
C.	Raised intracranial pressure
D.	Concussive syndrome
E.	Subarachnoid haemorrhage
F.	Migraine
G.	Tension headache
H.	Sinusitis
I.	Encephalitis
J.	Meningitis
K.	Brain abscess
L.	Trigeminal neuralgia
M.	Venous sinus thrombosis
N.	Severe hypertension
A

C. Raised intracranial pressure

Raised ICP classically presents with symptoms which are worse in the morning. The headache can either awake the patient from sleep or is present on waking up and decreases after being awake for several hours. It is also made worse on exertion/Valsalva such as the coughing and bending over here. This patient will require a CT head to try to find the cause. Raised ICP can also manifest with a CNVI palsy, which is a false localising sign.

How well did you know this?
1
Not at all
2
3
4
5
Perfectly
41
Q

Julie is a 20 year old student who is complaining of a pulsatile disabling right sided headache extending over the right eye. She noticed that the pain is worse in the evenings. She experienced a similar episode 3 months previously, which lasted for approximately 4 weeks. Her mother also suffers from headaches. Physical examination was unremarkable. Her BP was 120/80

Choose the most likely cause of the symptoms from the list of options.
A.	Analgesic rebound headache
B.	Temporal arteritis
C.	Raised intracranial pressure
D.	Concussive syndrome
E.	Subarachnoid haemorrhage
F.	Migraine
G.	Tension headache
H.	Sinusitis
I.	Encephalitis
J.	Meningitis
K.	Brain abscess
L.	Trigeminal neuralgia
M.	Venous sinus thrombosis
N.	Severe hypertension
A

F. Migraine

This headache is unilateral, pulsatile in quality and disabling. There is a FH and neurological examination is unremarkable, with normal BP. The patient has had a similar episode previously. Treatment of this chronic condition aims at treating acute attacks to restore function. Triptans can be used in specialist care. These are 5HT1 agonists. Effective initial treatment in a primary care setting can involve NSAIDs, which are available OTC. Treatment should be taken as soon as a patient realises they are having an attack and may need to be repeated after the attack. A few patients who have frequent, severe or disabling headaches may require daily prophylaxis such as anticonvulsants, TCAs and beta blockers.

How well did you know this?
1
Not at all
2
3
4
5
Perfectly
42
Q

A 60 year old lady presents with episodes of intense and sharp right sided pain in the mouth, running down to the jaw. She has had these attacks twice daily for 3 weeks now but has had these attacks for a few years now, although sporadically. She says eating sometimes brings on this pain.

Choose the most likely cause of the symptoms from the list of options.
A.	Analgesic rebound headache
B.	Temporal arteritis
C.	Raised intracranial pressure
D.	Concussive syndrome
E.	Subarachnoid haemorrhage
F.	Migraine
G.	Tension headache
H.	Sinusitis
I.	Encephalitis
J.	Meningitis
K.	Brain abscess
L.	Trigeminal neuralgia
M.	Venous sinus thrombosis
N.	Severe hypertension
A

L. Trigeminal neuralgia

Trigeminal neuralgia occurs as episodes of severe unilateral pain in the distribution of CNV, usually lasting seconds, with no pain occuring between these episodes. Examination is often unremarkable. The pain is described as sharp, intense, stabbing or burning. It can be triggered commonly by actions such as eating,tooth brushing, cold and touch. Shaving and eating seem to be common in EMQs. Most people are asymptomatic between attacks although the severity of the pain makes these patients live in constant fear. TN is more common in MS and incidence increases with age. Post-herpetic TN is also possible. The mainstay of treatment is medical, with antiepileptics such as carbamazepine (which is the only medicine which is proven in RCTs and is therefore typically first line). If medical treatment fails, surgical options do exist such as microvascular decompression.

How well did you know this?
1
Not at all
2
3
4
5
Perfectly
43
Q

A 50 year old man is stressed from work and has not been sleeping very well. He has had a few headaches and muscle aches in his arms and legs. As a result, he has been taking aspirin and paracetamol for most days of the past 4 months. He has now had a daily headache for the past week which is very painful. OTC drugs do not work, despite upping the dose. Examination is normal.

Choose the most likely cause of the symptoms from the list of options.
A.	Analgesic rebound headache
B.	Temporal arteritis
C.	Raised intracranial pressure
D.	Concussive syndrome
E.	Subarachnoid haemorrhage
F.	Migraine
G.	Tension headache
H.	Sinusitis
I.	Encephalitis
J.	Meningitis
K.	Brain abscess
L.	Trigeminal neuralgia
M.	Venous sinus thrombosis
N.	Severe hypertension
A

A. Analgesic rebound headache

This headache has resulted from medication overuse. There is a history of taking large amounts of over-the-counter analgesics. Examination is normal and the diagnosis is easily made on the history this patient gives. It may be difficult for the patient to accept that the pills he is taking for his sporadic headaches and aches have caused this severe daily headache he is now experiencing. Treatment involves stopping these OTC analgesics. This will often lead to worsening of the patient’s headache before it gets better and the patient needs to be warned about this.

How well did you know this?
1
Not at all
2
3
4
5
Perfectly
44
Q

A 67 year old woman suddenly develops a constant aching around her left eye after returning from the cinema. She complains of blurred vision in the affected eye. On examination, the left pupil was fixed & dilated & the entire eye was red. Fundoscopy revealed a cupped optic disc.

For each question choose the diagnosis from the list of options.
A.	Macular degeneration
B.	Horner’s syndrome
C.	Preproliferative retinopathy
D.	Extradural haematoma
E.	Migraine
F.	Diabetic background retinopathy
G.	Hypercholesterolaemia
H.	Oculomotor palsy
I.	Meningitis
J.	Conjuntivitis
K.	Paraneoplastic syndrome
L.	Acute glaucoma
M.	Subdural haemorrhage
A

L. Acute glaucoma

Acute closed-angle glaucoma can often present like this, with a change in vision with other severe acute symptoms such as eye pain, headache and N&V. This occurs when the iris comes into contact with the trabecular meshwork, which obstructs aqueous outflow from the eye such that intraocular pressure increases. This damages the optic nerve with loss of axons, leading to loss of visual field, which if untreated can progress to complete blindness. Typical changes that can be seen on fundoscopy include large optic disc cup. Immediate treatment is medical and aims at relieving symptoms and dropping IOP. Carbonic anhydrase inhibitors, topical beta blockers or alpha 2 adrenergic agents may be used, and typically in combination. In terms of pharmacology, CA inhibitors decrease aqueous humour formation. Hyper-osmotic agents are used when pressures are exceedingly high, or the patient is unresponsive to medical treatment. After the acute attack resolves, the patient should receive definitive surgical treatment within 24-48 hours to persistently create an open angle. This can be performed by laser peripheral iridotomy where a laser makes a hole in the iris to allow aqueous to bypass the pupil.

How well did you know this?
1
Not at all
2
3
4
5
Perfectly
45
Q

A 10 year old boy wakes to discover he cannot open his eyes because his lids are stuck together. When his mother prises them apart, his eyes are bright red & weeping & he cries that the sunlight is hurting them.

For each question choose the diagnosis from the list of options.
A.	Macular degeneration
B.	Horner’s syndrome
C.	Preproliferative retinopathy
D.	Extradural haematoma
E.	Migraine
F.	Diabetic background retinopathy
G.	Hypercholesterolaemia
H.	Oculomotor palsy
I.	Meningitis
J.	Conjuntivitis
K.	Paraneoplastic syndrome
L.	Acute glaucoma
M.	Subdural haemorrhage
A

J. Conjunctivitis

Acute conjunctivitis is most commonly allergic or infectious (bacterial or viral). Allergic is always bilateral and most commonly seasonal and associated with a history of asthma, hay fever, atopic dermatitis etc… Bacterial conjunctivitis is more likely to be bilateral than unilateral. Infectious conjunctivitis is associated with “gluing” of the eyelids. In sexually active young adults, gonorrhoeal and chlamydial conjunctivitis should be considered as a possible diagnosis. A viral cause (mostly adenovirus) typically starts in one eye and spreads to the other eye several days later, and can be associated with recent upper respiratory tract symptoms. Both bacterial and viral conjunctivitis are highly contagious so a contact history may be present. Patients can present with a red eye, itching, burning, a discharge and lymphadenopathy. Bacterial treatment includes topical antibiotics whereas viral conjunctivitis requires symptomatic treatment.

How well did you know this?
1
Not at all
2
3
4
5
Perfectly
46
Q

A 57 year old lorry driver attends a regular appointment with his optician. On fundoscopy, dots & blots are noted, in addition to hard exudates around the macula.

For each question choose the diagnosis from the list of options.
A.	Macular degeneration
B.	Horner’s syndrome
C.	Preproliferative retinopathy
D.	Extradural haematoma
E.	Migraine
F.	Diabetic background retinopathy
G.	Hypercholesterolaemia
H.	Oculomotor palsy
I.	Meningitis
J.	Conjuntivitis
K.	Paraneoplastic syndrome
L.	Acute glaucoma
M.	Subdural haemorrhage
A

F. Diabetic background retinopathy

This patient has a microvascular complication of diabetes – diabetic retinopathy. His macular involvement is an indication for macular laser therapy. Digital methods for photographing the fundus are preferred over fundoscopy. Microaneurysms, cotton wool spots, haemorrhages, hard/lipid exudates and neovascularisation may all be seen depending on the stage. Young onset T1DM is more associated with retinopathy, and additional risks include a longer duration of DM and poor glycaemic control compounded by renal disease and hypertension. There are also ethnic variations in risk. Background retinopathy initially consists of just microaneurysms, which progresses with small haemorrhages (dots and blots and flames). There may also be hard exudates. However, where these changes occur within the macula, this is maculopathy and is potentially sight-threatening. Maculopathy can be classified, academically, into focal, fissue and ischaemic, though this is of limited clinical relevance. Pre-proliferative is associated with cotton wool spots, typically multiple (arguably, the odd one can be consistent with background). Proliferative is associated with new vessel formation, which can be quite subtle. The patient can also be asymptomatic but is at a risk of vitreous haemorrhage. Diabetic retinopathy needs to be screened for (either dilated fundoscopy in clinic or with retinal photography) at least on an annual basis. Treatment is with photocoagulation where a transpupillary laser is used to burn the retina, which reduces overall oxygen demand lessening the ischaemic stimulus. There are complications, which you can look up. Please have a look at some photographs of retinal images. Reading about it will not help you to identify a cotton wool spot, hard exudate or angiogenesis in real life. There are also variations of normal which you will be able to spot with the more images you look at, like racial pigmentation and drusen in the elderly. What would you expect to see on fundoscopy in a patient with age-related macular degeneration?

How well did you know this?
1
Not at all
2
3
4
5
Perfectly
47
Q
An 89 year old male presents with his daughter, who has become worried about his gradual decline in physical &amp; intellectual ability over the past 8 weeks, since a fall at home. She says he complains of headaches &amp; seems sleepy. On examination, he has a left sided hemiparesis &amp; on fundoscopy, papilloedema is present.
For each question choose the diagnosis from the list of options.
A.	Macular degeneration
B.	Horner’s syndrome
C.	Preproliferative retinopathy
D.	Extradural haematoma
E.	Migraine
F.	Diabetic background retinopathy
G.	Hypercholesterolaemia
H.	Oculomotor palsy
I.	Meningitis
J.	Conjuntivitis
K.	Paraneoplastic syndrome
L.	Acute glaucoma
M.	Subdural haemorrhage
A

M. Subdural haemorrhage

A subdural occurs due to blood collecting between the dura mater and the arachnoid mater surrounding the brain. It may be arterial or venous although is most often venous. The disease course varies but this patient’s presentation fits. The cause is trauma – a fall in this case, most likely due to the patient’s elderly and frail state. Advanced age is associated with chronic subdurals. It is important in the examination to look for signs of trauma such as scalp abrasions and bruises. This patient is symptomatic and surgical options include twist-drill craniotomy with drainage (a bedside procedure where a hand drill is used to gain access to the subdural space and then a catheter is placed to act as a drain). Standard craniotomy is also an option, as is the creation of a burr hole. Remember that extradural haematomas classically have a ‘lucid interval’ and occur in younger patients, usually with an associated skull fracture, and CT of the haematoma does not cross suture lines.

How well did you know this?
1
Not at all
2
3
4
5
Perfectly
48
Q

A 46 year old lady, recently diagnosed with Pancoast’s syndrome showed the following facial signs: Miosis, enopthalmos & ptosis in her L eye & an ipsilateral absence of facial sweating.

For each question choose the diagnosis from the list of options.
A.	Macular degeneration
B.	Horner’s syndrome
C.	Preproliferative retinopathy
D.	Extradural haematoma
E.	Migraine
F.	Diabetic background retinopathy
G.	Hypercholesterolaemia
H.	Oculomotor palsy
I.	Meningitis
J.	Conjuntivitis
K.	Paraneoplastic syndrome
L.	Acute glaucoma
M.	Subdural haemorrhage
A

B. Horner’s syndrome

The presentation is of a pancoast tumour (most are non-small cell) in the left apex causing ipsilateral Horner’s syndrome (miosis, anhidrosis, ptosis and enophthalmos).

How well did you know this?
1
Not at all
2
3
4
5
Perfectly
49
Q

An 84 year old woman in a nursing home has been constipated for a week. Over the past few days she has become increasingly confused & incontinent.

For each presentation below, choose the single most discriminating investigation from the list of options.
A.	Chest x-ray
B.	Blood glucose
C.	Thyroid function tests
D.	Blood cultures
E.	Ultrasound abdomen
F.	Urea &amp; electrolytes
G.	Mid-stream specimen of urine
H.	CT scan of head
I.	Stool culture
J.	FBC
K.	ECG
A

G. Mid-stream specimen of urine

Constipation can be associated with a UTI and with this patient’s history, I would be highly suspicious of this diagnosis. An MSU sample and urine microscopy can confirm the organism type and guide antibiotic selection with culture and sensitivities of urine, which is the most specific and sensitive test for UTI. In real life, a urine dipstick would be the first test to order. Think about what you would expect the dipstick to show.

How well did you know this?
1
Not at all
2
3
4
5
Perfectly
50
Q

A previously well 78 year old woman has been noticed by her daughter to be increasingly slow & forgetful over several months. She has gained weight & tends to stay indoors with the heating on even in warm weather.

For each presentation below, choose the single most discriminating investigation from the list of options.
A.	Chest x-ray
B.	Blood glucose
C.	Thyroid function tests
D.	Blood cultures
E.	Ultrasound abdomen
F.	Urea &amp; electrolytes
G.	Mid-stream specimen of urine
H.	CT scan of head
I.	Stool culture
J.	FBC
K.	ECG
A

C. Thyroid function tests

This patient has symptoms of hypothyroidism. Worldwide, the most common cause is iodine deficiency. Other causes include Hashimoto’s or secondary and tertiary hypothyroidism. It can also result from viral de Quervain’s thyroiditis or postpartum thyroiditis. Diagnosis is based on measurement of TSH and thyroid hormones (thyroid function tests). Treatment is by replacement of T4 with or without T3 in combination.

How well did you know this?
1
Not at all
2
3
4
5
Perfectly
51
Q

A 64 year old man has recently been started on tablets by his GP. He is brought to the Accident & Emergency Department by his wife with sudden onset of aggressive behaviour, confusion & drowsiness. Prior to starting the tablets he was losing weight & complaining of thirst.

For each presentation below, choose the single most discriminating investigation from the list of options.
A.	Chest x-ray
B.	Blood glucose
C.	Thyroid function tests
D.	Blood cultures
E.	Ultrasound abdomen
F.	Urea &amp; electrolytes
G.	Mid-stream specimen of urine
H.	CT scan of head
I.	Stool culture
J.	FBC
K.	ECG
A

B. Blood glucose

This patient’s prior symptoms indicate he was diagnosed with DM and started on an oral hypoglycaemic, which can have the side effect of causing hypoglycaemia. Symptoms present when glucose drops <3mmol/L. Symptoms include sweating, weakness, drowsiness, palpitations and anxiety. This can be managed with oral glucose or sugar, or with IM glucagon. A blood glucose will demonstrate glucose levels consistent with his diagnosis.

How well did you know this?
1
Not at all
2
3
4
5
Perfectly
52
Q

A frail 85 year old woman presents with poor mobility & a recent history of falls. She has deteriorated generally over the past 2 weeks with fluctuating confusion. On examination she has a mild right hemiparesis.

For each presentation below, choose the single most discriminating investigation from the list of options.
A.	Chest x-ray
B.	Blood glucose
C.	Thyroid function tests
D.	Blood cultures
E.	Ultrasound abdomen
F.	Urea &amp; electrolytes
G.	Mid-stream specimen of urine
H.	CT scan of head
I.	Stool culture
J.	FBC
K.	ECG
A

H. CT scan of head

This is likely to be a subdural haematoma. A subdural occurs due to blood collecting between the dura mater and the arachnoid mater surrounding the brain. The cause is trauma – a fall in this case. A non-contrast CT will typically show a collection of subdural fluid in a crescenteric shape, which can cross suture lines.

How well did you know this?
1
Not at all
2
3
4
5
Perfectly
53
Q

A 75 year old man with known mild Alzheimer’s disease suddenly fainted. When seen in the Accident & Emergency Department, his blood pressure was 90/60 & his pulse rate was 40/min & regular.

For each presentation below, choose the single most discriminating investigation from the list of options.
A.	Chest x-ray
B.	Blood glucose
C.	Thyroid function tests
D.	Blood cultures
E.	Ultrasound abdomen
F.	Urea &amp; electrolytes
G.	Mid-stream specimen of urine
H.	CT scan of head
I.	Stool culture
J.	FBC
K.	ECG
A

K. ECG

This patient is bradycardic and hypotensive. The low blood pressure is a manifestation of low cardiac output due to bradycardia. This patient needs an ECG to investigate the rhythm. The syncope could be due to an arryhthmia of some sort – for example, complete heart block, which resulted in cerebral hypoperfusion. The Alzheimer’s is of no real relevance here.

How well did you know this?
1
Not at all
2
3
4
5
Perfectly
54
Q

An unconscious 35 year old man who has a capillary blood glucose of 1.5mmol/l.

For each of the cases below, choose the most appropriate next management option from the list.
A.	Commence CPR
B.	Gastric lavage
C.	CT scan brain
D.	Inhaled anticholinergic
E.	Intravenous naloxone
F.	Precordial thump
G.	Intravenous dextrose
H.	DC cardioversion
I.	Intravenous antibiotics
J.	Lumbar puncture
K.	Endotracheal intubation
L.	IM glucagon
A

G. Intravenous dextrose

This patient is profoundly hypoglycaemic (symptoms of hypoglycaemia are present when glucose drops <3mmol/L). Patients with either long standing DM or on beta blockers may become unaware of hypoglycaemia and become profoundly hypoglycaemic before symptoms develop. In DM, hypoglycaemia is usually secondary to insulin or oral hypoglycaemics. Non DM causes of hypoglycaemia include insulinomas, alcohol, liver failure and Addison’s disease. Treatment is corrective and for someone this profoundly hypoglycaemic, IV dextrose is needed. Care should be taken when administering such high % dextrose IV due to the risk of skin necrosis if administered incorrectly (if the IV leaks). IM glucagon is only used if IV access cannot be established.

How well did you know this?
1
Not at all
2
3
4
5
Perfectly
55
Q

A 55 year old man found collapsed at home who, on arrival at hospital, has no palpable pulse or recordable blood pressure.

For each of the cases below, choose the most appropriate next management option from the list.
A.	Commence CPR
B.	Gastric lavage
C.	CT scan brain
D.	Inhaled anticholinergic
E.	Intravenous naloxone
F.	Precordial thump
G.	Intravenous dextrose
H.	DC cardioversion
I.	Intravenous antibiotics
J.	Lumbar puncture
K.	Endotracheal intubation
L.	IM glucagon
A

A. Commence CPR

This patient has cardiac arrest and the first thing you need to do is commence CPR. It is thought to work by raising intrathoracic pressure and providing direct cardiac compression. 30 compressions (at 100/min) and 2 breaths for a total of 5 cycles (2 minutes), makes up one cycle of CPR. Compressions are the first priority, breaths may follow but compressions are paramount. Further management depends on whether the patient has a shockable rhythm (pulseless CT or VF) or non-shockable rhythm (PEA or asystole). Do you know when a precordial thump can be used?

How well did you know this?
1
Not at all
2
3
4
5
Perfectly
56
Q

An 18 year old woman found unconscious at home. She has needle “track” marks in her arms, a respiratory rate of 10/min & pinpoint pupils.

For each of the cases below, choose the most appropriate next management option from the list.
A.	Commence CPR
B.	Gastric lavage
C.	CT scan brain
D.	Inhaled anticholinergic
E.	Intravenous naloxone
F.	Precordial thump
G.	Intravenous dextrose
H.	DC cardioversion
I.	Intravenous antibiotics
J.	Lumbar puncture
K.	Endotracheal intubation
L.	IM glucagon
A

E. Intravenous nalaxone

Opiate OD signs include CNS depression, miosis and apnoea. Finding small constricted pupils in someone who is unconscious is highly indicative. Naloxone is indicated both therapeutically and diagnostically. If there is a response, then it is diagnostic. Another diagnosis should be sought if the patient is unresponsive. IV is the preferred route of administration although naloxone can be given IM or SC if IV access cannot be established. Ventilatory support is key with 100% oxygen. You can check out Toxbase for a full database on poisons and treatments.

How well did you know this?
1
Not at all
2
3
4
5
Perfectly
57
Q

A 34 year old woman who complained of a severe headache on waking & then collapsed.

For each of the cases below, choose the most appropriate next management option from the list.
A.	Commence CPR
B.	Gastric lavage
C.	CT scan brain
D.	Inhaled anticholinergic
E.	Intravenous naloxone
F.	Precordial thump
G.	Intravenous dextrose
H.	DC cardioversion
I.	Intravenous antibiotics
J.	Lumbar puncture
K.	Endotracheal intubation
L.	IM glucagon
A

C. CT scan brain

A CT head is indicated here in this possible SAH. This may show hyperdense areas in the basal cisterns, major fissures and sulci.

How well did you know this?
1
Not at all
2
3
4
5
Perfectly
58
Q

An 18 year old known asthmatic presents with a respiratory rate of 50 and inaudible breath sounds on auscultation. The patient is cyanosed and ABG reports a respiratory acidosis. The patient has not responded to medication.

For each of the cases below, choose the most appropriate next management option from the list.
A.	Commence CPR
B.	Gastric lavage
C.	CT scan brain
D.	Inhaled anticholinergic
E.	Intravenous naloxone
F.	Precordial thump
G.	Intravenous dextrose
H.	DC cardioversion
I.	Intravenous antibiotics
J.	Lumbar puncture
K.	Endotracheal intubation
L.	IM glucagon
A

K. Endotracheal intubation

This is life threatening asthma that has not responded to therapy. There is cyanosis and respiratory acidosis despite the tachypnoea of 50/min. This patient is clearly too dyspnoeic to speak and oxygen saturation may be quite low. Pulsus paradoxus may also be observed. This patient should be intubated (mechanical ventilation is required) and transfered to ICU and given supplemental oxygen. IV corticosteroids should also be used and heliox therapy considered (helium-oxygen).

59
Q

A 70 year old man with pain in the jaw after chewing food. He has also noticed pains in his shoulder & pelvic area over the past 3-4 weeks, plus an episode of transient visual loss in the right eye.

For each patient select the most likely cause of their headache.
A.	Encephalitis
B.	Analgesic rebound headache
C.	Tension headache
D.	Subarachnoid haemorrhage
E.	Severe hypotension
F.	Meningitis
G.	Raised intracranial pressure
H.	Migrane
I.	Trigeminal neuralgia
J.	Sinusitis
K.	Temporal arteritis
A

K. Temporal arteritis

Temporal arteritis/GCA is a form of vasculitis which commonly affects those >50 and females. This normally presents with a headache typically over the temporal/occipital areas and may be accompanied by scalp tenderness. While this patient does not complain of a headache, his other symptoms still give a high index of suspicion. There may, like in this case,be PMR symptoms such as aching and stiffness of the proximal extremities worse after inactivity and with movement (there is an association with PMR). This patient also demonstrates jaw claudication and the artery may be thickened and tender on examination. This patient also describes amaurosis fugax which can be a vision symptom of temporal arteritis. Systemic symptoms like a low grade fever and fatigue are commonly seen. Treatment is with oral prednisolone which should be started if there is high suspicion of this diagnosis and this should not even be delayed by results of ESR/CRP. IV pulse methylprednisolone is preferred in this case due to the presence of visual symptoms. The best test to establish a diagnosis is with a temporal artery biopsy, however this should not delay treatment as irreversible blindness is the most common serious consequence of this condition. The arteritis is patchy so an adequate biopsy sample is needed. Histopathology will show a granulomatous inflammatory process, with giant cells present in around half of cases. GCA is unlikely (although 5% occur with normal ESR) if inflammatory markers are normal so an ESR is a quick test and >100 predicts a positive biopsy result (likelihood ratio of 1.9).

60
Q

A 15 year old girl presented to her GP with headache and a rash. On examination she was pyrexial and there was restricted neck movement and photophobia. She has a purpuric rash on her legs which did not fade on pressure.

For each patient select the most likely cause of their headache.
A.	Encephalitis
B.	Analgesic rebound headache
C.	Tension headache
D.	Subarachnoid haemorrhage
E.	Severe hypotension
F.	Meningitis
G.	Raised intracranial pressure
H.	Migrane
I.	Trigeminal neuralgia
J.	Sinusitis
K.	Temporal arteritis
A

F. Meningitis

This patient has meningitis. Students are commonly affected due to crowding. Commonly there will be a headache, fever and nuchal rigidity. There may also be an altered mental status, confusion, photophobia and vomiting. Kernig’s sign is uncommon but is positive when attempts to extend the leg are met with resistance when the patient is supine with the thigh flexed to 90 degrees. Another uncommon sign is Brudzinski’s sign and a petechial/purpuric rash, typically associated with meningococcal meningitis. CT head should be considered before LP if there is any evidence of raised ICP. An LP will confirm the diagnosis with bacterial meningitis showing a low CSF glucose, elevated CSF protein and positive CSF culture/gram stain or meningococcal antigen.

61
Q

A 15 month old toddler developed a maculo-papular rash 10 days after receiving the MMR vaccine, followed by a swelling at the angle of the left jaw. His mother phoned the surgery in a state of panic one morning because he is confused, having convulsions and has a high temperature.

For each patient select the most likely cause of their headache.
A.	Encephalitis
B.	Analgesic rebound headache
C.	Tension headache
D.	Subarachnoid haemorrhage
E.	Severe hypotension
F.	Meningitis
G.	Raised intracranial pressure
H.	Migrane
I.	Trigeminal neuralgia
J.	Sinusitis
K.	Temporal arteritis
A

A. Enephalitis

Encephalitis (brain parenchyma inflammation) is not to be confused with meningitis where the meninges is inflammed (although meningoencephalitis can exist). In encephalitis, there is an altered state of consciousness and there may also be focal neurology. There is also a fever (exceptions, however, are subacute sclerosing panencephalitis, VZV and HCV). Other typical features include headache and seizures. It is a medical emergency and people are the extremes of age are more at risk. The list of possible causes is massive but in this case, vaccination seems to be the cause. Any vaccination such as MMR can be associated with developing ADEM (acute disseminated encephalomyelitis). The maculopapular rash and the parotitis seen in this patient allude to mumps. However, children not vaccinated against mumps and measles are at risk of mumps or measles encephalitis, and there is also a risk of subacute sclerosing panencephalitis from measles. (Overall, it is still much safer to be vaccinated). Treatment of ADEM is with high dose corticosteroids and supportive care (circulatory and electrolyte support, possible ET intubation, ventilation etc…). If corticosteroids fail to work, plasma exchange (consult your friendly haematologist) or IV immunoglobulin can be considered.

62
Q

A 30 year old man presented at the local A&E dept. with severe headaches. He described them as explosions inside his head. Whilst lying on the hospital trolley he suddenly loses conciousness. The CSF sample came back negative for any organisms but the SHO who performed the lumbar puncture commented on blood in the sample.

For each patient select the most likely cause of their headache.
A.	Encephalitis
B.	Analgesic rebound headache
C.	Tension headache
D.	Subarachnoid haemorrhage
E.	Severe hypotension
F.	Meningitis
G.	Raised intracranial pressure
H.	Migrane
I.	Trigeminal neuralgia
J.	Sinusitis
K.	Temporal arteritis
A

D. Subarachnoid haemorrhage

An LP showing bloody CSF (xanthochromia or gross blood) suggests SAH. It is worth noting that xanthochromia is absent in the first 12 hours after SAH. SAH (bleeding into the subarachnoid space) presents with sudden severe headache patients will often describe as the worst headache of their life, and can often be so bad that they feel like they’ve been kicked in the back of the back. Half of all patients lose consciousness and eye pain with exposure to light can also be seen. Altered mental status is common. SAH occurs most commonly in the 50-55 age group and affects women and black people more than men and white people. The most common cause of non-traumatic SAH is an aneurysm which ruptures. Conditions which predispose to aneurysm formation and SAH include adult PKD, Marfan’s, NF1 and Ehlers-Danlos. Cerebral aneurysms arise around the circle of Willis. A CT scan is indicated, and if unrevealing, this should be followed by an LP. Cerebral angiography can confirm the presence of aneurysms. The patient should be stabilised and this followed by surgical clipping or endovascular coil embolisation, the choice is subject to much current controversy sparked by relatively recent research. Complications can commonly occur and include rebleeding, hydrocephalus and vasospasm.

63
Q

A 40 year old teacher presented with a 24 hour history of occipital headache, which became worse in the evenings after work. Her school is due for an Ofsted inspection shortly. On examination she is apyrexial but looks ill and is tachycardic (120bpm)

For each patient select the most likely cause of their headache.
A.	Encephalitis
B.	Analgesic rebound headache
C.	Tension headache
D.	Subarachnoid haemorrhage
E.	Severe hypotension
F.	Meningitis
G.	Raised intracranial pressure
H.	Migrane
I.	Trigeminal neuralgia
J.	Sinusitis
K.	Temporal arteritis
A

C. Tension headache

A tension headache is commonly triggered by stress and mental tension (also, fatigure and missing meals), hence the name. It is more common in females and those in middle age, and there is a link with lower socioeconomic status, although this does not necessarily represent causation. Symptoms include a dull, non-pulsatile and constricting bilateral pain, which is often described as a band across the patient’s head. It is not severe or disabling but classically worsens as the day progresses. This headache normally responds well to simple analgesics.

64
Q

A 50 year old man with type I diabetes feels light headed on standing. He has had 4 episodes of collapse with transient loss of consciousness after standing up.

For each of the following patients choose the SINGLE most discriminatory investigation from the list of options.
A. Carotid sinus massage
B. Tilt test
C. Prolonged fasting blood sugar
D. CT brain
E. Carotid doppler
F. 24 hour Holter monitor
G. Exercise ECG
H. ECG
I. EEG
A

B. Tilt test

This patient has postural hypotension, demonstrated by a fall of >20 in systolic blood pressure and >10 diastolic within 3 minutes of standing upright. This is sufficient to be diagnostic, but a tilt-table test can also be used. In this test, beat-to-beat recordings of BP and HR reveal an immediate and progressive decline in BP with a small increase in HR on head-up tilt. This is due to impaired autonomic reflexes, in this man, due to diabetic autonomic neuropathy. HR and BP are continuously measured while the patient is supine and during head-up tilt.

65
Q

A 60 year old man has had 3 episodes of loss of consciousness without warning. A witness said that he went very pale as he collapsed, but then became flushed & regained consciousness after 30 seconds. He vomited after each collapse. Resting pulse 60 beats/min.

For each of the following patients choose the SINGLE most discriminatory investigation from the list of options.
A. Carotid sinus massage
B. Tilt test
C. Prolonged fasting blood sugar
D. CT brain
E. Carotid doppler
F. 24 hour Holter monitor
G. Exercise ECG
H. ECG
I. EEG
A

F. 24 hour Holter monitor

An ECG while the patient is asymptomatic would not pick up the problem, so a 24 hour continuous ambulatory (Holter) ECG is needed so the rhythm when the patient has the attack can be analysed. This is helpful in patients with events which occur several times a week. If the symptoms are less frequent, an event or wearable loop recorder may be more useful. The history this patient gives is consistent with a Stokes-Adams attack). Stokes-Adams attacks (cardiogenic syncope) are episodes of transient LOC due to sudden decreased cardiac output. Pallor prior to the attack and facial flushing due to reactive hyperemia after the attack is characteristic of a Stokes-Adams attack. Definitive treatment is with surgical insertion of a pacemaker. Holter may end up showing evidence of complete AV block.

66
Q

A 60 year old woman has recently started attending an exercise class. She has collapsed whilst exercising on 3 occasions & has also experienced chest pain during exercise. Pulse 60/min, BP 100/80.

For each of the following patients choose the SINGLE most discriminatory investigation from the list of options.
A. Carotid sinus massage
B. Tilt test
C. Prolonged fasting blood sugar
D. CT brain
E. Carotid doppler
F. 24 hour Holter monitor
G. Exercise ECG
H. ECG
I. EEG
A

G. Exercise ECG

The patient’s history suggests a cardiac cause of her symptoms (you may be thinking an arrhythmia or outlet obstruction like aortic stenosis). An exercise stress ECG could show onset of symptoms, ST changes, an abnormal BP response or arrhythmia.

67
Q

A 35 year old woman has a 20 year history of infrequent episodes where she feels nauseated, with a rising feeling in her epigastrium & chest, 20 seconds later she collapses & is unconscious for up to a minute. During this she wrings her hands but is not incontinent & has never injured herself. Afterwards she feels tired for about 2 hours.

For each of the following patients choose the SINGLE most discriminatory investigation from the list of options.
A. Carotid sinus massage
B. Tilt test
C. Prolonged fasting blood sugar
D. CT brain
E. Carotid doppler
F. 24 hour Holter monitor
G. Exercise ECG
H. ECG
I. EEG
A

I. EEG

This is a focal seizure (arises from one portion of the brain) characterised by movement of one specific part of this patient’s body consistent with localised brain activity. It is associated most often with the temporal lobe. There may be, like this case, a premonitory sensation such as in the epigastrium. EEG may show spikes or sharp waves in one temporal lobe, however, history taking is still the most important aspect of diagnosis (although admittedly can be difficult to obtain). All patients with a suspected seizure should have an EEG but it is not necessarily diagnostic. Many patients also complain of feeling tired after a seizure. An antiepileptic agent is indicated and the patient should be started off on monotherapy of a drug like carbamazepine.

68
Q

A 65 year old man has had 3 collapses in the last week. He has no warning & sustained a black eye in the last one. He feels giddy all the time. Resting pulse is 40 beats/min.

For each of the following patients choose the SINGLE most discriminatory investigation from the list of options.
A. Carotid sinus massage
B. Tilt test
C. Prolonged fasting blood sugar
D. CT brain
E. Carotid doppler
F. 24 hour Holter monitor
G. Exercise ECG
H. ECG
I. EEG
A

H. ECG

His bradycardia and collapse needs investigating and an ECG will reveal the rhythm of his bradycardia and perhaps give us an idea as to what is causing his collapse. It is worth noting that carotid sinus massage is useful if you suspect a diagnosis of carotid sinus hypersensitivity.

69
Q

A 72 year old man with uncontrolled blood sugar has presented with ulcers in his right foot & bilateral loss of sensation below knee.

Match the following clinical scenarios with the most likely diagnosis from the list.
A.	TIA
B.	Diabetic neuropathy
C.	Left sided stroke
D.	Bell’s palsy
E.	Subarachnoid haemorrhage
F.	Right sided stroke
G.	Motor neuron disease
H.	Multiple sclerosis
I.	Parkinson’s disease
J.	Meningitis
K.	Subdural haemorrhage
L.	Epilepsy
A

B Diabetic neuropathy

Diabetic neuropathy has lead to the painless ulcer most likely developing over pressure points in the foot such as on the ball of the foot. This may have resulted from an object becoming lodged in the shoe and eroding through the skin with walking. This is why it is important for diabetics to check their feet regularly and to wear specialised footwear. Diabetic neuropathy is a microvascular complication of DM and is characterised by peripheral nerve dysfunction. Pain is a common complaint such as the burning sensation this patient describes. Patient’s may also describe the pain as prickling or sticking.

Complications range from the painless neuropathic ulcer described, at areas of the foot where there is weight loading (particularly the metatarsal heads), to the Charcot foot with severe architectural destruction of the foot. Foot ulceration is a common precusor to amputation. Foot care is crucial in DM. Examination should include peripheral pulses, reflexes and sensation to light touch with a 10g monofilament, vibration (128Hz tuning fork), pinprick and proprioception. The pain may be treated with medications like pregabalin and gabapentin.

70
Q

A 77 year old lady with 15 year history of hypertension is brought to A&E. She is unable to speak & cannot move her right arm & leg.

Match the following clinical scenarios with the most likely diagnosis from the list.
A.	TIA
B.	Diabetic neuropathy
C.	Left sided stroke
D.	Bell’s palsy
E.	Subarachnoid haemorrhage
F.	Right sided stroke
G.	Motor neuron disease
H.	Multiple sclerosis
I.	Parkinson’s disease
J.	Meningitis
K.	Subdural haemorrhage
L.	Epilepsy
A

C. Left sided stroke

Hemiplegia and the inability to speak makes this likely to be a stroke. If you have a think about the motor pathways you will realise that this is a left sided stroke. It is important is perform a CT head exclude a haemorrhagic aetiology and consider thrombolysis with tPA if within the 4.5 hour window and there are no contraindications. Thrombolysis is done with alteplase at 10% bolus, 90% infusion at a dose of 0.9 mg/kg. Presentation after the 4.5 hour window is managed with aspirin. The Bamford/Oxford Stroke Classification subtypes ischaemic stroke according to vascular territory of infarction. After initial management, stroke care involves the ethos of an MDT environment with rehabilitation.

71
Q

A 17 year old boy comes to the clinic with his mother. He complains of recurrent transient episodes of tingling & weakness in his left leg. His mother comments that he has also been a bit clumsy and occasionally he has had trouble speaking clearly. On direct questioning he admits some blurred vision, but “it comes and goes”.

Match the following clinical scenarios with the most likely diagnosis from the list.
A.	TIA
B.	Diabetic neuropathy
C.	Left sided stroke
D.	Bell’s palsy
E.	Subarachnoid haemorrhage
F.	Right sided stroke
G.	Motor neuron disease
H.	Multiple sclerosis
I.	Parkinson’s disease
J.	Meningitis
K.	Subdural haemorrhage
L.	Epilepsy
A

H. Multiple sclerosis

Multiple sclerosis is a demyelinating CNS condition which is characterised by 2 or more episodes of neurological dysfunction which are separated in both time and space. This person has had blurred vision and now 6 months later has developed facial palsy. A RAPD is seen in a Marcus Gunn pupil which is tested for with the swinging light test in a CN examination and can be caused by any disease affecting the optic nerve. MS classically presents in white women aged 20-40 with temporary visual/sensory loss although any presentation can occur. MRI is a sensitive test but less specific than spinal MRI, however, spinal MRI is abnormal in fewer cases. Treatment aims at treating the attack, preventing future attacks and symptomatic treatment of problems like bladder dysfunction, pain and fatigue.

72
Q

A 79 year old man complains of difficulty walking. On examination you also notice he has a resting tremor & his limbs oppose movement.

Match the following clinical scenarios with the most likely diagnosis from the list.
A.	TIA
B.	Diabetic neuropathy
C.	Left sided stroke
D.	Bell’s palsy
E.	Subarachnoid haemorrhage
F.	Right sided stroke
G.	Motor neuron disease
H.	Multiple sclerosis
I.	Parkinson’s disease
J.	Meningitis
K.	Subdural haemorrhage
L.	Epilepsy
A

I. Parkinson’s disease

Parkinson’s is characterised by a resting tremor, rigidity, bradykinesia and postural instability. This patient has difficulty walking, a resting tremor (4-6 Hz at rest which dissipates with the use of limbs, with generally asymmetrical onset) and limbs oppose movement. This last point demonstrates rigidity, which shows as resistance to passive movement about a joint. There is often also cogwheeling, especially if there is a superimposed tremor. The patient may have other signs like a mask like face due to the loss of spontaneous facial movement, hypophonia and micrographia, and may walk around in a shuffling gait with a stooped posture. The diagnosis is clinical. Treatment is symptomatic in an MDT setting. Medical therapy includes MAO-B inhibitors and DA agonists, for example rasagiline and carbidopa/levodopa. There are other medical therapies depending on the specific symptoms the patient presents with.

73
Q

A 50 year old man develops sudden onset of weakness, numbness & paraesthesiae on the left side of his body. His symptoms faded gradually before disappearing 12 hours later.

Match the following clinical scenarios with the most likely diagnosis from the list.
A.	TIA
B.	Diabetic neuropathy
C.	Left sided stroke
D.	Bell’s palsy
E.	Subarachnoid haemorrhage
F.	Right sided stroke
G.	Motor neuron disease
H.	Multiple sclerosis
I.	Parkinson’s disease
J.	Meningitis
K.	Subdural haemorrhage
L.	Epilepsy
A

A. TIA

A TIA is colloquially called a ‘mini stroke’ with symptoms typically lasting under an hour (and resolve within 24 hours). An antiplatelet drug such as aspirin is effective secondary prevention if the patient is not already anticoagulated. The patient will be anticoagulated if they have a likely or known cardioembolic source such as AF. Clopidogrel is an alternative in those who do not tolerate aspirin.

74
Q

A 40 year old man with sudden onset of a severe headache, drowsiness & vomiting.

Match the following clinical scenarios with the most likely diagnosis from the list.
A.	TIA
B.	Diabetic neuropathy
C.	Left sided stroke
D.	Bell’s palsy
E.	Subarachnoid haemorrhage
F.	Right sided stroke
G.	Motor neuron disease
H.	Multiple sclerosis
I.	Parkinson’s disease
J.	Meningitis
K.	Subdural haemorrhage
L.	Epilepsy
A

E. Subarachnoid haemorrhage

SAH (bleeding into the subarachnoid space) presents with sudden severe headache patients will often describe as the worst headache of their life, and can often be so bad that they feel like they’ve been kicked in the back of the back. Half of all patients lose consciousness and eye pain with exposure to light can also be seen. Altered mental status is common. SAH occurs most commonly in the 50-55 age group and affects women and black people more than men and white people. The most common cause of non-traumatic SAH is an aneurysm which ruptures. Conditions which predispose to aneurysm formation and SAH include adult PKD, Marfan’s, NF1 and Ehlers-Danlos. Cerebral aneurysms arise around the circle of Willis. A CT scan is indicated, and if unrevealing, this should be followed by an LP. Cerebral angiography can confirm the presence of aneurysms. The patient should be stabilised and this followed by surgical clipping or endovascular coil embolisation, the choice is subject to much current controversy sparked by relatively recent research. Complications can commonly occur and include rebleeding, hydrocephalus and vasospasm.

75
Q

A 60 year old man had a fall following one of his drunken episodes. During the following 2 weeks he experienced intermittent periods of drowsiness & headaches. His wife also observed some changes in his personality notably aggressive behaviour.

For each patient below, choose the most likely cause of the symptom from the list of options.
A.	Carpal tunnel syndrome
B.	TIA
C.	Parkinson’s disease
D.	Pontine haemorrhage
E.	Hepatic encephalopathy
F.	PCA aneurysm
G.	Bell’s palsy
H.	Meningitis
I.	Vestibular schwannoma
J.	Subarachnoid haemorrhage
K.	Myasthenia gravis
L.	Huntington's disease
M.	Multiple sclerosis
N.	Epilepsy
O.	Subdural haemorrhage
A

O. Subdural haemorrhage

A subdural occurs due to blood collecting between the dura mater and the arachnoid mater surrounding the brain. It may be arterial or venous although is most often venous. The disease course varies. There is neurological deficit evident so surgery will be indicated. The cause is trauma from the fall this man has had. Advanced age is associated with chronic subdurals. It is important in the examination to look for signs of trauma such as scalp abrasions and bruises. This patient is symptomatic and surgical options include twist-drill craniotomy with drainage (a bedside procedure where a hand drill is used to gain access to the subdural space and then a catheter is placed to act as a drain). Standard craniotomy is also an option, as is the creation of a burr hole. Remember that extradural haematomas classically have a ‘lucid interval’ and occur in younger patients, usually with an associated skull fracture, and CT of the haematoma does not cross suture lines. A non-contrast CT will typically show a collection of subdural fluid in a crescenteric shape, which can cross suture lines.

76
Q

A 20 year old student developed a headache, fever, & vomiting during one of his lectures. A class mate reports that he was breathing very quickly. On arrival at the surgery 30 minutes later he was semi-conscious, & breathing irregularly. His upper limbs were also jerking.

For each patient below, choose the most likely cause of the symptom from the list of options.
A.	Carpal tunnel syndrome
B.	TIA
C.	Parkinson’s disease
D.	Pontine haemorrhage
E.	Hepatic encephalopathy
F.	PCA aneurysm
G.	Bell’s palsy
H.	Meningitis
I.	Vestibular schwannoma
J.	Subarachnoid haemorrhage
K.	Myasthenia gravis
L.	Huntington's disease
M.	Multiple sclerosis
N.	Epilepsy
O.	Subdural haemorrhage
A

H. Meningitis

This patient has meningitis. Meningitis can present with a host of symptoms and commonly presents with seizures (children who have seizures are more likely to be infected by Streptococcus pneumoniae and Haemophilus influenza than meningococcus). Universities are common sites of outbreaks due to crowding. Commonly there will be a headache, fever and nuchal rigidity. There may also be an altered mental status, confusion, photophobia and vomiting. Kernig’s sign is uncommon but is positive when attempts to extend the leg are met with resistance when the patient is supine with the thigh flexed to 90 degrees. Another uncommon sign is Brudzinski’s sign and a petechial/purpuric rash, typically associated with meningococcal meningitis.

CT head should be considered before LP if there is any evidence of raised ICP. An LP will confirm the diagnosis with bacterial meningitis showing a low CSF glucose, elevated CSF protein and positive CSF culture/gram stain or meningococcal antigen.

77
Q

A 30 year old woman experienced a strange feeling in her stomach, followed by stiffness & jerking in the left arm. She was incontinent of urine before becoming drowsy & falling asleep.

For each patient below, choose the most likely cause of the symptom from the list of options.
A.	Carpal tunnel syndrome
B.	TIA
C.	Parkinson’s disease
D.	Pontine haemorrhage
E.	Hepatic encephalopathy
F.	PCA aneurysm
G.	Bell’s palsy
H.	Meningitis
I.	Vestibular schwannoma
J.	Subarachnoid haemorrhage
K.	Myasthenia gravis
L.	Huntington's disease
M.	Multiple sclerosis
N.	Epilepsy
O.	Subdural haemorrhage
A

N. Epilepsy

The feeling this woman felt in her stomach before (aura), the description of the arm, incontinence and feeling “worn out” afterwards make this a seizure. Seizures can happen in anyone but epilepsy is the tendency to have repeated, unprovoked seizures (but this is still the best answer option from the available list). Epileptic seizures can be split into focal (partial/localised) and generalised (involving both hemispheres). Generalised seizures can be further divided into abscence (which can be typical or atypical), myoclonic, clonic, tonic, tonic-clonic and atonic types. Be sure to exclude other causes, such as a space occupying lesion with a CT or MRI scan. There may also be tongue biting,. Always think of “before, during and after” when taking the history. History taking is the most important aspect of diagnosis. Antiepileptic monotherapy is the preferred treatment.

78
Q

A 50 year old man with left sided facial weakness, watering of the left eye & dribbling. On examination he was unable to close his left eye or whistle.

For each patient below, choose the most likely cause of the symptom from the list of options.
A.	Carpal tunnel syndrome
B.	TIA
C.	Parkinson’s disease
D.	Pontine haemorrhage
E.	Hepatic encephalopathy
F.	PCA aneurysm
G.	Bell’s palsy
H.	Meningitis
I.	Vestibular schwannoma
J.	Subarachnoid haemorrhage
K.	Myasthenia gravis
L.	Huntington's disease
M.	Multiple sclerosis
N.	Epilepsy
O.	Subdural haemorrhage
A

G. Bell’s palsy

Bell’s palsy is idiopathic unilateral LMN CNVII palsy (hence, this as yet uninvestigated caase is best described as a CNVII palsy). It is a diagnosis of exclusion as the aetiology is unknown. Corticosteroids are effective and surgical decompression can be considered in severe cases. Eye protection should be considered too. This condition tends to resolve itself by 4-6 months. Think about what you’d expect to find on examination of CNVII.

Think back to anatomy. CNVII supplies all the muscles of facial expression, stapedius and some other muscles (posterior belly of digastric, stylohyoid and occipitofrontalis). It is sensory to the external auditory meatus (via the nervus intermedius). It is parasympathetic via the same nerve, to supply the submandibular and sublingual glands. The same nerve also carries taste sensation in the anterior two thirds of the tongue, and innervates the palate. Do you still remember the branches of the facial nerve?

79
Q

A 40 year old lady with pain & tingling in the left hand, worse at night time when she had to get out of bed to shake the hand for relief.

For each patient below, choose the most likely cause of the symptom from the list of options.
A.	Carpal tunnel syndrome
B.	TIA
C.	Parkinson’s disease
D.	Pontine haemorrhage
E.	Hepatic encephalopathy
F.	PCA aneurysm
G.	Bell’s palsy
H.	Meningitis
I.	Vestibular schwannoma
J.	Subarachnoid haemorrhage
K.	Myasthenia gravis
L.	Huntington's disease
M.	Multiple sclerosis
N.	Epilepsy
O.	Subdural haemorrhage
A

A. Carpal tunnel syndrome

Carpal tunnel syndrome is the most common nerve entrapment and women just past middle age are at the highest risk. Symptoms include numbness/tingling of the thumb and radial fingers, an aching wrist and clumsiness (especially with fine motor tasks). The symptoms are of gradual onset and often wake the patient up at night, and is relieved by shaking the wrist. Numbness is normally on the palmar aspect of the thumb, index and middle fingers (but not the little finger). When the patient wakes up, there may be difficulty flexing or extending fingers. Symptoms in the day tend to be associated with activity. The most sensitive and specific test for diagnosis is EMG and can confirm damage to the median nerve in the carpal tunnel and categorise the severity of the damage. There are specific tests for CTS such as Tinel’s test and Phalen’s test, though clinically these are not particularly useful due to sensitivity and specificity.

CTS is caused by anything that causes a reduction in the size of the carpal tunnel – from inflammation, arthritis and tenosynovitis to old fractures. In CTS there is preserved sensation of the palm as the palmar cutaneous branch comes off a few cm above the carpal tunnel.

Remember your anatomy in 2nd year? The median nerve lies in the carpal tunnel, deep to the flexor retinaculum and in the concavity bounded by the carpal bones. Other structures lying in the carpal tunnel include FPL, FDS and FDP.

80
Q

A 25 year old pregnant lady with increasing muscle weakness. She also complains of double vision & drooping eye lids.

For each patient below, choose the most likely cause of the symptom from the list of options.
A.	Carpal tunnel syndrome
B.	TIA
C.	Parkinson’s disease
D.	Pontine haemorrhage
E.	Hepatic encephalopathy
F.	PCA aneurysm
G.	Bell’s palsy
H.	Meningitis
I.	Vestibular schwannoma
J.	Subarachnoid haemorrhage
K.	Myasthenia gravis
L.	Huntington's disease
M.	Multiple sclerosis
N.	Epilepsy
O.	Subdural haemorrhage
A

K. Myasthenia gravis

Myasthenia gravis is an autoimmune condition with antibodies affecting the NMJ, mostly the nAChR at the post-synaptic muscle membrane. Although some have antibodies against MuSK, and there are other proteins involved. MG is characterised by muscle weakness which increases with exercise (fatigue, unlike Lambert-Eaton myasthenic syndrome). Commonly, presentations include diplopia and drooping eyelids like this patient, and there may also be SOB, proximal limb weakness, facial paresis and oropharyngeal weakness. MG is associated with thymic hyperplasia in 70% or thymoma in 10%, and these associations can also crop up in EMQs. There will be elevated serum AChR receptor antibody titres or MuSK antibodies. Electrophysiology will demonstrate a decremental response on repetitive nerve stimulation. Treatment includes anticholinesterases (pyridostigmine, and immunotherapy. Patients may also require a thymectomy. Some 15-20% may experience a myasthenic crisis (which needs mechanical ventilation). Do you know what the Tensilon test is and why edrophonium is given in this test?

81
Q

The wife of a 60 year old man noticed that his movements had slowed down recently. His gait is shuffling and he had a resting tremor of his hands.

For each patient below, choose the most likely cause of the symptom from the list of options.
A.	Carpal tunnel syndrome
B.	TIA
C.	Parkinson’s disease
D.	Pontine haemorrhage
E.	Hepatic encephalopathy
F.	PCA aneurysm
G.	Bell’s palsy
H.	Meningitis
I.	Vestibular schwannoma
J.	Subarachnoid haemorrhage
K.	Myasthenia gravis
L.	Huntington's disease
M.	Multiple sclerosis
N.	Epilepsy
O.	Subdural haemorrhage
A

C. Parkinson’s disease

Parkinson’s is characterised by a resting tremor, rigidity, bradykinesia and postural instability. The resting tremor occurs at 4-6 Hz at rest which dissipates with the use of limbs, with generally asymmetrical onset. There is often also cogwheeling, especially if there is a superimposed tremor. The patient may have other signs like a mask like face due to the loss of spontaneous facial movement, hypophonia and micrographia, and may walk around in a shuffling gait with a stooped posture. The diagnosis is clinical. Treatment is symptomatic in an MDT setting. Medical therapy includes MAO-B inhibitors and DA agonists, for example rasagiline and carbidopa/levodopa. There are other medical therapies depending on the specific symptoms the patient presents with.

82
Q

A 30 year old lady presented with pain in her left eye and numbness & weakness of her right leg. 2 months earlier she had an episode of double vision in the left eye.

For each patient below, choose the most likely cause of the symptom from the list of options.
A.	Carpal tunnel syndrome
B.	TIA
C.	Parkinson’s disease
D.	Pontine haemorrhage
E.	Hepatic encephalopathy
F.	PCA aneurysm
G.	Bell’s palsy
H.	Meningitis
I.	Vestibular schwannoma
J.	Subarachnoid haemorrhage
K.	Myasthenia gravis
L.	Huntington's disease
M.	Multiple sclerosis
N.	Epilepsy
O.	Subdural haemorrhage
A

M. Multiple sclerosis

Multiple sclerosis is a demyelinating CNS condition which is characterised by 2 or more episodes of neurological dysfunction which are separated in both time and space. MS classically presents in white women aged 20-40 with temporary visual/sensory loss although any presentation can occur. MRI is a sensitive test but less specific than spinal MRI, however, spinal MRI is abnormal in fewer cases. Treatment aims at treating the attack, preventing future attacks and symptomatic treatment of problems like bladder dysfunction, pain and fatigue.

83
Q

A 52 year old fund manager with a history of previous heart attacks, feels some palpitations and collapses. . A witness said that he went very pale as he collapsed but then became flushed and regained consciousness after 30 seconds.

For each question, choose the single most appropriate diagnosis from the list of options.
A.	Pulmonary embolism
B.	Hypoglycemia
C.	Postural hypotension
D.	Anaemia
E.	TIA
F.	Stokes-Adams attack
G.	Stroke
H.	Epilepsy
I.	UTI
J.	Vasovagal syncope
A

F. Stokes-Adams attack

The history this patient gives is consistent with a Stokes-Adams attack). Stokes-Adams attacks (cardiogenic syncope) are episodes of transient LOC due to sudden decreased cardiac output. Pallor prior to the attack and facial flushing due to reactive hyperemia after the attack is characteristic of a Stokes-Adams attack. Definitive treatment is with surgical insertion of a pacemaker. Cause may be complete AV block

84
Q

A 24 year old diabetic starts a new preparation of insulin. He chases after his bus, missing it and collapses on the street.

For each question, choose the single most appropriate diagnosis from the list of options.
A.	Pulmonary embolism
B.	Hypoglycemia
C.	Postural hypotension
D.	Anaemia
E.	TIA
F.	Stokes-Adams attack
G.	Stroke
H.	Epilepsy
I.	UTI
J.	Vasovagal syncope
A

B. Hypoglycaemia

In DM, hypoglycaemia is usually secondary to insulin or oral hypoglycaemics. Symptoms of hypoglycaemia are present when glucose drops <3mmol/Land this patient’s clinical state has been exacerbated by his new insulin preparation and him running for his bus. Patients with either long standing DM or on beta blockers may become unaware of hypoglycaemia and become profoundly hypoglycaemic before symptoms develop. How would you treat this patient?

85
Q

An 80 year old man fainted with a 2 week history of abdominal pain and coughing up a black coffee-ground like substance. He has been feeling irritable, tired and sleepy.

For each question, choose the single most appropriate diagnosis from the list of options.
A.	Pulmonary embolism
B.	Hypoglycemia
C.	Postural hypotension
D.	Anaemia
E.	TIA
F.	Stokes-Adams attack
G.	Stroke
H.	Epilepsy
I.	UTI
J.	Vasovagal syncope
A

D. Anaemia

This man is anaemic and as a result he has fainted. This is IDA from a UGI bleed. His faint can also be attributed to hypovolaemia from his blood loss, from a presumed peptic ulcer (which accounts for his abdominal pain and coffee ground vomit). Whilst he may also display postural hypotension due to his hypovolaemia, this is not what this question is looking for.

86
Q

A 75 year old man is found on his bedroom floor by his wife and is conscious. He got out of bed in the middle of the night to go to the toilet and felt dizzy and fell to the ground. He is on treatment for hypertension and has no other medical problems.

For each question, choose the single most appropriate diagnosis from the list of options.
A.	Pulmonary embolism
B.	Hypoglycemia
C.	Postural hypotension
D.	Anaemia
E.	TIA
F.	Stokes-Adams attack
G.	Stroke
H.	Epilepsy
I.	UTI
J.	Vasovagal syncope
A

C. Postural hypotension

This patient has postural hypotension which is demonstrated by a fall of >20 in systolic blood pressure and >10 diastolic within 3 minutes of standing upright. It is a side effect of anti-hypertensives and is a common problem in the elderly. A good history should be enough to diagnose this.

87
Q

A 25-year-old man was hit on the side of the head with the ball, whilst playing cricket. He recovered enough to finish off the game. He then presented to A&E 8 hours later with a severe headache and vomiting. He rapidly loses consciousness.

What is the most likely diagnosis?
A. Cluster headache
B. Migraine
C. Subarachnoid haemorrhage
D. Extradural haemorrhage
E. Trigeminal neuralgia
F. Tension headache
A

D. Extradural haemorrhage

Here we have the ‘lucid interval’ classically associated with an extradural haematoma. There is blood buildup this time between the dura mater and the skull. Compressive signs may also be present such as the down and out pupil due to CN III compression. The bleed here is usually from arteries, under high pressure, causing raised intracranial pressure. In this case, there is a chance the brain stem has been compressed causing his LOC. The bleed here is possibly from the anterior branch of the middle meningeal artery which runs beneath the pterion and is vulnerable at this point where the skull is thin. Remember that the pterion marks the junction of four bones: the parietal bone, temporal bone, sphenoid bone and frontal bone.

88
Q

A 20-year-old medical student complains of a throbbing pain bilaterally across her forehead, which is present for some hours each time. She is busy catching up on online EMQs and lectures for her exams next week. Physical examination is normal.

What is the most likely diagnosis?
A. Cluster headache
B. Migraine
C. Subarachnoid haemorrhage
D. Extradural haemorrhage
E. Trigeminal neuralgia
F. Tension headache
A

F. Tension headache

A tension headache is commonly triggered by stress and mental tension (also, fatigure and missing meals), hence the name. It is more common in females and those in middle age, and there is a link with lower socioeconomic status, although this does not necessarily represent causation. Symptoms include a dull, non-pulsatile and constricting bilateral pain, which is often described as a band across the patient’s head. It is not severe or disabling but classically worsens as the day progresses. This headache normally responds well to simple analgesics.

89
Q

A 20-year-old student complains of a throbbing headache. It lasts for a few hours, and is associated with nausea. She can usually predict the headache half an hour before, when she sees flashing lights.

What is the most likely diagnosis?
A. Cluster headache
B. Migraine
C. Subarachnoid haemorrhage
D. Extradural haemorrhage
E. Trigeminal neuralgia
F. Tension headache
A

B. Migraine

Migraine is a chronic condition, with genetic determinants, which usually presents in early to mid life. The typical migraine aura this patient describes (which can be visual, sensory or speech symptoms) which can occur during or before the headache, is pathognomic, but is not seen in the majority of patients. The aura can be positive phenomena (for example this patient seeing flashing lights) or negative phenomena (for example visual loss). Nausea, photophobia and disability (the headache gets in the way with the patient’s ability to function) accompanying a headache also suggest a migraine diagnosis. The headache of a migraine tends to be prolonged if untreated, and tends to be unilateral and pounding (but does not have to be). Tests aim to rule out other differentials, although if the history is compatible and neurological examination is unremarkable, further testing is not needed.

90
Q

A 34-year-old man has attacks of sudden severe pain waking him up for the last fortnight. The pain is on the right side of his face and makes his right eye water.

What is the most likely diagnosis?
A. Cluster headache
B. Migraine
C. Subarachnoid haemorrhage
D. Extradural haemorrhage
E. Trigeminal neuralgia
F. Tension headache
A

A. Cluster headache

Cluster headache is characterised by attacks of severe pain localised to the unilateral orbital, supraorbital and/or temporal areas which lasts from 15 minutes to 3 hours, and occurs with a frequency ranging from once every other day to 8 times a day. These attacks can occur at the same time period of many weeks (known as the cluster period) accompanied by ipsilateral autonomic signs. The cause is hypothalamic activation with secondary trigeminal and autonomic activation (for instance, lacrimation, rhinorrhoea, nasal congestion, conjunctival injection and partial Horner’s i.e. ptosis and miosis). Cluster period attacks can be triggered by things like alcohol. Greater occipital nerve blockade often provides immediate relief until preventative medications take effect.

91
Q

A 56-year-old man has a sudden pain that ‘shoots’ to his chin whilst shaving. It disappears after a few seconds.

What is the most likely diagnosis?
A. Cluster headache
B. Migraine
C. Subarachnoid haemorrhage
D. Extradural haemorrhage
E. Trigeminal neuralgia
F. Tension headache
A

E. Trigeminal neuralgia

Trigeminal neuralgia occurs as episodes of severe unilateral pain in the distribution of CNV, usually lasting seconds, with no pain occurring between these episodes. Examination is often unremarkable. The pain is described as sharp, intense, stabbing or burning. It can be triggered commonly by actions such as eating, tooth brushing, cold and touch. Shaving and eating seem to be common in EMQs. Most people are asymptomatic between attacks although the severity of the pain makes these patients live in constant fear. TN is more common in MS and incidence increases with age. Post-herpetic TN is also possible. The mainstay of treatment is medical, with antiepileptics such as carbamazepine (which is the only medicine which is proven in RCTs and is therefore typically first line). If medical treatment fails, surgical options do exist such as microvascular decompression.

92
Q

A 40-year-old man presents with an instantaneous onset of a severe headache, followed by drowsiness & vomiting.

What is the most likely diagnosis?
A. Cluster headache
B. Migraine
C. Subarachnoid haemorrhage
D. Extradural haemorrhage
E. Trigeminal neuralgia
F. Tension headache
A

C. Subarachnoid haemorrhage

SAH (bleeding into the subarachnoid space) presents with sudden severe headache patients will often describe as the worst headache of their life, and can often be so bad that they feel like they’ve been kicked in the back of the back. Half of all patients lose consciousness and eye pain with exposure to light can also be seen. Altered mental status is common. SAH occurs most commonly in the 50-55 age group and affects women and black people more than men and white people.

The most common cause of non-traumatic SAH is an aneurysm which ruptures. Conditions which predispose to aneurysm formation and SAH include adult PKD, Marfan’s, NF1 and Ehlers-Danlos. Cerebral aneurysms arise around the circle of Willis. A CT scan is indicated, and if unrevealing, this should be followed by an LP. Cerebral angiography can confirm the presence of aneurysms. The patient should be stabilised and this followed by surgical clipping or endovascular coil embolisation, the choice is subject to much current controversy sparked by relatively recent research. Complications can commonly occur and include rebleeding, hydrocephalus and vasospasm.

93
Q

A 52 year old fund manager with a history of previous heart attacks, feels some palpitations and collapses. A witness said that he went very pale as he collapsed but then became flushed and regained consciousness after 30 seconds.

What is the most likely diagnosis?
A. Opioid overdose
B. Stokes-Adams attack
C. Postural hypotension
D. Hypoglycaemia
E. Anaemia
A

B. Stokes-Adams attack

Stokes-Adams attacks are episodes of transient LOC due to sudden decreased cardiac output. The previous heart attacks and later palpitations towards an arrhythmia such as heart bock, which caused the attack. Pallor prior to the attack and facial flushing due to reactive hyperaemia after the attack is characteristic of a Stokes-Adams attack. Definitive treatment is with surgical insertion of a pacemaker.

94
Q

A 24-year-old patient with diabetes starts a new preparation of insulin. After going to the gym he feels sweaty, nauseous and dizzy. A few minutes later he is found unconscious on the floor.

What is the most likely diagnosis?
A. Opioid overdose
B. Stokes-Adams attack
C. Postural hypotension
D. Hypoglycaemia
E. Anaemia
A

D. Hypoglycaemia

In DM, hypoglycaemia is usually secondary to insulin or oral hypoglycaemics. Symptoms of hypoglycaemia are present when glucose drops <3mmol/Land this patient’s clinical state has been exacerbated by his new insulin preparation and him running for his bus. Patients with either long standing DM or on beta blockers may become unaware of hypoglycaemia and become profoundly hypoglycaemic before symptoms develop. How would you treat this patient?

95
Q

An 80 year old man fainted with a 2 week history of abdominal pain and coughing up a black coffee-ground like substance. He has been feeling irritable, tired and sleepy.

What is the most likely diagnosis?
A. Opioid overdose
B. Stokes-Adams attack
C. Postural hypotension
D. Hypoglycaemia
E. Anaemia
A

E. Anaemia

This man is anaemic and as a result he has fainted. This is IDA from a UGI bleed. His faint can also be attributed to hypovolaemia from his blood loss, from a presumed peptic ulcer (which accounts for his abdominal pain and coffee ground vomit). Whilst he may also display postural hypotension due to his hypovolaemia, this is not what this question is looking for.

96
Q

A 75-year-old man is found on his bedroom floor by his wife and is now conscious. He got out of bed in the middle of the night to go to the toilet and felt dizzy and fell to the ground. He is on treatment for hypertension and has no other medical problems.

What is the most likely diagnosis?
A. Opioid overdose
B. Stokes-Adams attack
C. Postural hypotension
D. Hypoglycaemia
E. Anaemia
A

C. Postural hypotension

This patient has postural hypotension which is demonstrated by a fall of >20 in systolic blood pressure and >10 diastolic within 3 minutes of standing upright. It is a side effect of anti-hypertensives and is a common problem in the elderly. A good history should be enough to diagnose this.

97
Q

A 38-year-old homeless man is found unconscious on the street. On examination he has pinpoint pupils, a respiratory rate of 6 breaths per minute and needle marks on both arms.

What is the most likely diagnosis?
A. Opioid overdose
B. Stokes-Adams attack
C. Postural hypotension
D. Hypoglycaemia
E. Anaemia
A

A. Opioid overdose

Opiate OD signs include CNS depression, miosis and apnoea. Finding small constricted pupils in someone who is unconscious is highly indicative. Naloxone is indicated both therapeutically and diagnostically. If there is a response, then it is diagnostic. Another diagnosis should be sought if the patient is unresponsive. IV is the preferred route of administration although naloxone can be given IM or SC if IV access cannot be established. Ventilatory support is key with 100% oxygen. You can check out Toxbase for a full database on poisons and treatments.

98
Q

A 20-year-old feverish student developed a headache and vomiting during his sociology lecture. The only other student who attended, reports that he was breathing very quickly. On arrival at the surgery 30 minutes later he was semi-conscious, & breathing irregularly. His upper limbs were also jerking.

What is the most likely diagnosis?
A. Parkinson's disease
B. Partial seizure
C. PCA aneurysm
D. Pontine haemorrhage
E. Multiple sclerosis
F. Huntington’s disease
G. Hepatic encephalopathy
H. Right-sided stroke
I. Meningitis
J. Myasthenia gravis
K. Transient ischaemic attack
A

I. Meningitis

This 20 year old student has meningitis. Meningitis can present with a host of symptoms and commonly presents with seizures (children who have seizures are more likely to be infected by Streptococcus pneumoniae and Haemophilus influenza than meningococcus). Universities are common sites of outbreaks due to crowding. Commonly there will be a headache, fever and nuchal rigidity. There may also be an altered mental status, confusion, photophobia and vomiting. Kernig’s sign is uncommon but is positive when attempts to extend the leg are met with resistance when the patient is supine with the thigh flexed to 90 degrees. Another uncommon sign is Brudzinski’s sign and a petechial/purpuric rash, typically associated with meningococcal meningitis.

CT head should be considered before LP if there is any evidence of raised ICP. An LP will confirm the diagnosis with bacterial meningitis showing a low CSF glucose, elevated CSF protein and positive CSF culture/gram stain or meningococcal antigen.

99
Q

A 30-year-old woman experienced a strange feeling in her stomach, followed by stiffness & jerking in the left arm. Afterwards, she felt drowsy but remembers everything.

What is the most likely diagnosis?
A. Parkinson's disease
B. Partial seizure
C. PCA aneurysm
D. Pontine haemorrhage
E. Multiple sclerosis
F. Huntington’s disease
G. Hepatic encephalopathy
H. Right-sided stroke
I. Meningitis
J. Myasthenia gravis
K. Transient ischaemic attack
A

B. Partial seizure

The feeling this woman felt in her stomach before (aura), the description of the arm, incontinence and feeling “worn out” afterwards make this a seizure. Seizures can happen in anyone but epilepsy is the tendency to have repeated, unprovoked seizures (but this is still the best answer option from the available list). Epileptic seizures can be split into focal (partial/localised) and generalised (involving both hemispheres). Generalised seizures can be further divided into absence (which can be typical or atypical), myoclonic, clonic, tonic, tonic-clonic and atonic types. Be sure to exclude other causes, such as a space occupying lesion with a CT or MRI scan. There may also be tongue biting. Always think of “before, during and after” when taking the history. History taking is the most important aspect of diagnosis. Antiepileptic monotherapy is the preferred treatment.

100
Q

An 80-year-old woman who is a smoker was brought into A&E from a residential home where her carers noticed that she had difficulty swallowing and that she also had difficulty moving her left arm and leg for the past few days.

What is the most likely diagnosis?
A. Parkinson's disease
B. Partial seizure
C. PCA aneurysm
D. Pontine haemorrhage
E. Multiple sclerosis
F. Huntington’s disease
G. Hepatic encephalopathy
H. Right-sided stroke
I. Meningitis
J. Myasthenia gravis
K. Transient ischaemic attack
A

H. Right-sided stroke

Weakness on one side and the difficulty swallowing makes this likely to be a stroke. If you have a think about the motor pathways you will realise that this is a right sided stroke. It is important is perform a CT head to exclude a haemorrhagic aetiology and consider thrombolysis with tPA if within the 4.5 hour window and there are no contraindications. Thrombolysis is done with alteplase at 10% bolus, 90% infusion at a dose of 0.9 mg/kg. Presentation after the 4.5 hour window is managed with aspirin. The Bamford/Oxford Stroke Classification subtypes ischaemic stroke according to vascular territory of infarction. After initial management, stroke care involves the ethos of an MDT environment with rehabilitation.

101
Q

A 25-year-old pregnant lady presents with increasing muscle weakness. She also complains of double vision & drooping eye lids.

What is the most likely diagnosis?
A. Parkinson's disease
B. Partial seizure
C. PCA aneurysm
D. Pontine haemorrhage
E. Multiple sclerosis
F. Huntington’s disease
G. Hepatic encephalopathy
H. Right-sided stroke
I. Meningitis
J. Myasthenia gravis
K. Transient ischaemic attack
A

J. Myasthenia gravis

This lady has myasthenia gravis, which is an autoimmune condition with antibodies affecting the NMJ, mostly the nAChR at the post-synaptic muscle membrane. Although some have antibodies against MuSK, and there are other proteins involved. MG is characterised by muscle weakness which increases with exercise (fatigue, unlike Lambert-Eaton myasthenic syndrome). Commonly, presentations include diplopia and drooping eyelids like this patient, and there may also be SOB, proximal limb weakness, facial paresis and oropharyngeal weakness. MG is associated with thymic hyperplasia in 70% or thymoma in 10%, and these associations can also crop up in EMQs. There will be elevated serum AChR receptor antibody titres or MuSK antibodies. Electrophysiology will demonstrate a decremental response on repetitive nerve stimulation. Treatment includes anticholinesterases (pyridostigmine, and immunotherapy. Patients may also require a thymectomy. Some 15-20% may experience a myasthenic crisis (which needs mechanical ventilation). Do you know what the Tensilon test is and why edrophonium is given in this test?

102
Q

A 79-year-old man complains of difficulty walking. On examination you also notice he has a resting tremor & his limbs oppose movement.

What is the most likely diagnosis?
A. Parkinson's disease
B. Partial seizure
C. PCA aneurysm
D. Pontine haemorrhage
E. Multiple sclerosis
F. Huntington’s disease
G. Hepatic encephalopathy
H. Right-sided stroke
I. Meningitis
J. Myasthenia gravis
K. Transient ischaemic attack
A

A. Parkinson’s disease

Parkinson’s is characterised by a resting tremor, rigidity, bradykinesia and postural instability. The resting tremor occurs at 4-6 Hz at rest which dissipates with the use of limbs, with generally asymmetrical onset. There is often also cogwheeling, especially if there is a superimposed tremor. The patient may have other signs like a mask like face due to the loss of spontaneous facial movement, hypophonia and micrographia, and may walk around in a shuffling gait with a stooped posture. The diagnosis is clinical. Treatment is symptomatic in an MDT setting. Medical therapy includes MAO-B inhibitors and DA agonists, for example rasagiline and carbidopa/levodopa. There are other medical therapies depending on the specific symptoms the patient presents with.

103
Q

A 30-year-old lady presented with pain in her left eye and numbness & weakness of her right leg. Two months earlier she had an episode of double vision in the left eye.

What is the most likely diagnosis?
A. Parkinson's disease
B. Partial seizure
C. PCA aneurysm
D. Pontine haemorrhage
E. Multiple sclerosis
F. Huntington’s disease
G. Hepatic encephalopathy
H. Right-sided stroke
I. Meningitis
J. Myasthenia gravis
K. Transient ischaemic attack
A

E. Multiple sclerosis

Multiple sclerosis is a demyelinating CNS condition which is characterised by 2 or more episodes of neurological dysfunction which are separated in both time and space. MS classically presents in white women aged 20-40 with temporary visual/sensory loss although any presentation can occur. MRI is a sensitive test but less specific than spinal MRI, however, spinal MRI is abnormal in fewer cases. Treatment aims at treating the attack, preventing future attacks and symptomatic treatment of problems like bladder dysfunction, pain and fatigue.

104
Q

A 65-year-old hypertensive man has complained of losing vision twice in one eye, which lasted for a few hours and then went back to normal. He says it’s like a ‘a black sheet falling over the front of my eye’.

What is the most likely diagnosis?
A. Parkinson's disease
B. Partial seizure
C. PCA aneurysm
D. Pontine haemorrhage
E. Multiple sclerosis
F. Huntington’s disease
G. Hepatic encephalopathy
H. Right-sided stroke
I. Meningitis
J. Myasthenia gravis
K. Transient ischaemic attack
A

K. Transient ischaemic attack

A TIA is colloquially called a ‘mini stroke’ with symptoms typically lasting under an hour. Here there is a description of focal neurological deficit unilaterally in the form of amaurosis fugax. There may also be other signs like unilateral weakness or sensory loss, hemianopsia, aphasia and cranial nerve defects, to name but a few. An antiplatelet drug such as aspirin is effective secondary prevention if the patient is not already anticoagulated. The patient will be anticoagulated if they have a likely or known cardioembolic source such as AF. Clopidogrel is an alternative in those who do not tolerate aspirin.

105
Q

A 72-year-old man with a HbA1C of 12.2% has presented with ulcers in his right foot & bilateral loss of sensation below knee.

What is the most likely diagnosis?
A. Carpal tunnel syndrome
B. Diabetic neuropathy
C. Charcot-Marie-Tooth disease

A

B. Diabetic neuropathy

Diabetic neuropathy has lead to the painless ulcer most likely developing over pressure points in the foot such as on the ball of the foot. This may have resulted from an object becoming lodged in the shoe and eroding through the skin with walking. This is why it is important for diabetics to check their feet regularly and to wear specialised footwear. Diabetic neuropathy is a microvascular complication of DM and is characterised by peripheral nerve dysfunction. Pain is a common complaint such as the burning sensation this patient describes. Patient’s may also describe the pain as prickling or sticking.

Complications range from the painless neuropathic ulcer described, at areas of the foot where there is weight loading (particularly the metatarsal heads), to the Charcot foot with severe architectural destruction of the foot. Foot ulceration is a common precusor to amputation. Foot care is crucial in DM. Examination should include peripheral pulses, reflexes and sensation to light touch with a 10g monofilament, vibration (128Hz tuning fork), pinprick and proprioception. The pain may be treated with medications like pregabalin and gabapentin.

106
Q

A 40-year-old typist presents with pain & tingling in the lateral side of the left hand, worse at night when she had to get out of bed to shake the hand for relief.

What is the most likely diagnosis?
A. Carpal tunnel syndrome
B. Diabetic neuropathy
C. Charcot-Marie-Tooth disease

A

A. Carpal tunnel syndrome

Carpal tunnel syndrome is the most common nerve entrapment and women just past middle age are at the highest risk. Symptoms include numbness/tingling of the thumb and radial fingers, an aching wrist and clumsiness (especially with fine motor tasks). The symptoms are of gradual onset and often wake the patient up at night, and is relieved by shaking the wrist. Numbness is normally on the palmar aspect of the thumb, index and middle fingers (but not the little finger). When the patient wakes up, there may be difficulty flexing or extending fingers. Symptoms in the day tend to be associated with activity. The most sensitive and specific test for diagnosis is EMG and can confirm damage to the median nerve in the carpal tunnel and categorise the severity of the damage. There are specific tests for CTS such as Tinel’s test and Phalen’s test, though clinically these are not particularly useful due to sensitivity and specificity.

CTS is caused by anything that causes a reduction in the size of the carpal tunnel – from inflammation, arthritis and tenosynovitis to old fractures. In CTS there is preserved sensation of the palm as the palmar cutaneous branch comes off a few cm above the carpal tunnel.

107
Q

A 10-year-old boy has wasting of his lower leg, which look like ‘inverted champagne bottles’. His father had the same condition.

What is the most likely diagnosis?
A. Carpal tunnel syndrome
B. Diabetic neuropathy
C. Charcot-Marie-Tooth disease

A

C. Charcot-Marie-Tooth disease

Charcot-Marie-Tooth disease comprises of a group of hereditary peripheral neuropathies with different genetic abnormalities, hence the FH here (most patients have a FHx). Pes cavus (high foot arches), hammer toes and distal atrophy of the hands and legs are characteristic (resulting in this inverted champagne bottle appearance). Pes cavus can be associated with areflexia. Patients may have to lift their legs up excessively to clear the toes. Rehabilitative and orthotic treatments can help to keep patients functional. There are no other known risk factors for this condition aside from FH. The risk of passing the condition on to the next generation varies depending on the subtype of CMT, which have different inheritance patterns. Most CMT1 and CMT2 are autosomal dominant and CMT 4 is autosomal recessive. There are also X-linked forms.

108
Q

A 75 year old lady is brought to A&E by her daughter who found her on the floor. She appears confused which you confirm with 4/10 on the AMTS. O/E she is febrile and complains of a burning pain in her lower abdomen.

What is the most likely diagnosis?
A. U&amp;E imbalance
B. Post-operative
C. Hypoxia
D. CVA
E. Drug effect
F. Liver failure
G. Thiamine/B1 deficiency
H. Faecal impaction
I. MI
J. Hypoglycaemia.
K. UTI
A

K. UTI

Confusion and a fever is not an uncommon presentation of UTI in the elderly where the presentation is often non-specific. The lower abdominal suprapubic pain here is another clue. 10 percent of women over the age of 70 have UTIs. Post-menopause (absence of oestrogen) is a strong risk factor for UTI in women. In women of this age group, sexual activity is less strongly associated with UTI. Treatment can be started on the symptoms here as well as dipstick result, but urine culture and sensitivity confirms the diagnosis and guides the appropriate use of antibiotics. A positive nitrite and leucocyte esterase in the urine indicates a likely UTI diagnosis. A midstream clean-catch urine specimen should be sent for culture (looking for uropathogenic organisms) if symptoms are atypical or do not respond to treatment.

The AMTS is used to rapidly assess mental function and a score of 6 or less suggests either delirium or dementia (the former is acute decline). Do you know how to perform an AMTS screen?

109
Q

A 42 year old man who lives alone presents with confusion and ataxia. Both his hands have a Dupytren’s contracture.

What is the most likely diagnosis?
A. U&amp;E imbalance
B. Post-operative
C. Hypoxia
D. CVA
E. Drug effect
F. Liver failure
G. Thiamine/B1 deficiency
H. Faecal impaction
I. MI
J. Hypoglycaemia.
K. UTI
A

G. Thiamine/B1 deficiency

This patient has Wernicke’s encephalopathy. Wernicke’s is due to acute thiamine deficiency, which is a problem in alcoholics (Dupytren’s contracture here is a clue to alcohol use). Others at risk include those with AIDS, cancer and treatment with chemotherapy, malnutrition and GIT surgery, especially bariatric procedures. It is a clinically under-diagnosed condition. The classic EMQ triad is of mental change, ophthalmoplegia and gait dysfunction, which is actually only seen in 10% of cases. In reality, the manifestations are varied and a high index of suspicion is needed. Despite there, the manifestations typically include altered consciousness, gait disorders and eye movement abnormalities. This is an emergency and treatment is with parenteral replacement of thiamine. This avoids permanent neurological damage including later development of Korsakoff’s psychosis, which is irreversible. Note that thiamine should be given before dextrose! Magnesium deficiency also needs to be corrected as it is a co-factor in the functioning of thiamine dependent enzymes.

110
Q

A 16 year old boy dressed in sports kit, presents to A&E confused, sweating and with a tremor. His speech starts to slur and he begins to lose consciousness.

What is the most likely diagnosis?
A. U&amp;E imbalance
B. Post-operative
C. Hypoxia
D. CVA
E. Drug effect
F. Liver failure
G. Thiamine/B1 deficiency
H. Faecal impaction
I. MI
J. Hypoglycaemia.
K. UTI
A

J. Hypoglycaemia.

This patient has symptoms of hypoglycaemia, present when glucose drops <3mmol/L. Symptoms include sweating, weakness, drowsiness, palpitations and anxiety. In more severe cases, confusion and LOC can occur. Mild cases can be managed with oral glucose or sugar. Patients with either long standing DM or on beta blockers may become unaware of hypoglycaemia and become profoundly hypoglycaemic before symptoms develop. In DM, hypoglycaemia is usually secondary to insulin or oral hypoglycaemics. Non DM causes of hypoglycaemia include insulinomas, alcohol, liver failure and Addison’s disease. The sports kit here indicates that exercise may be a precipitating factor in this case.

Treatment is corrective and for someone this profoundly hypoglycaemic, IV dextrose is needed. Care should be taken when administering such high % dextrose IV due to the risk of skin necrosis if administered incorrectly (if the IV leaks). IM glucagon is only used if IV access cannot be established.

111
Q

An 80 year old man on your ward becomes confused. He has no signs of an infection. You notice his mouth and lips are dry.

What is the most likely diagnosis?
A. U&amp;E imbalance
B. Post-operative
C. Hypoxia
D. CVA
E. Drug effect
F. Liver failure
G. Thiamine/B1 deficiency
H. Faecal impaction
I. MI
J. Hypoglycaemia.
K. UTI
A

A. U&E imbalance

This patient has become confused and there is an acute change in mental status. Dehydration (volume depletion) can be a relatively common cause. (The most common causes of volume depletion are haemorrhage, vomiting, diarrhoea, diuresis or third space losses). FBC in this case may reveal a raised haematocrit and high Hb. Serum electrolyte analysis may reveal derangements such as hyper or hypokalaemia and hyponatraemia. Urine specific gravity can be expected to be high and serum urea to creatinine ratio >20. Examination of this dehydrated patient may reveal signs like dry mucosal membranes, postural hypotension, postural tachycardia and possibly even shock depending on severity. The patient may complain of symptoms such as thirst, tiredness, muscle cramps, pain in the chest and/or abdomen as well as confusion. The confusion may result from U&E abnormalities (the best option from this list) but is non-specific and may indeed reflect poor cerebral perfusion or uraemia in the setting of impaired renal function, which are equally if not more likely in this patient.

112
Q

A 77 year old lady with longstanding AF presents with confusion. She collapsed suddenly at home. O/E she has an extensor plantar response.

What is the most likely diagnosis?
A. U&amp;E imbalance
B. Post-operative
C. Hypoxia
D. CVA
E. Drug effect
F. Liver failure
G. Thiamine/B1 deficiency
H. Faecal impaction
I. MI
J. Hypoglycaemia.
K. UTI
A

D. CVA

Atrial fibrillation is strongly implicated in the risk of cardioembolic stroke. Confusion is common, especially in the elderly who have had past strokes and in those with cognitive dysfunction. An extensor plantar response is known as Babinski’s sign and is a sign of an UMN lesion. The normal response is flexion where the big toe moves downwards. It is important is perform a CT head exclude a haemorrhagic aetiology and consider thrombolysis with tPA if within the 4.5 hour window and there are no contraindications. Thrombolysis is done with alteplase at 10% bolus, 90% infusion at a dose of 0.9 mg/kg. Presentation after the 4.5 hour window is managed with aspirin. The Bamford/Oxford Stroke Classification subtypes ischaemic stroke according to vascular territory of infarction. After initial management, stroke care involves the ethos of an MDT environment with rehabilitation.

113
Q

A 65 year old man with a history of an MI 2 years ago. He lost consciousness and presents to you 36 hours later with reduced power in his left arm and leg.

What is the most likely diagnosis?
A. Epilepsy
B. Vasovagal syncope
C. Cardiac arrhythmia
D. Stroke
E. Postural hypotension
F. TIA
G. Aortic stenosis
H. Anaemia
I. Hypoglycaemia
J. Pulmonary stenosis
K. Myxoedema coma
L. Carotid sinus sensitivity
A

D. Stroke

If you have a think about the motor pathways you will realise that this is a right sided stroke. Cardiac conditions are potential risks for cardioembolism. The previous MI could, for instance, lead to regional wall motion abnormalities or decreased LV ejection fraction which can be risks. Other cardiac conditions include valve disease, PFO, mitral prolapse, prosthetic heart valve and cardiomyopathy. Other major risk factors include AF, DM, smoking, hypertension and FH. It is important is perform a CT head exclude a haemorrhagic aetiology and consider thrombolysis with tPA if within the 4.5 hour window and there are no contraindications. Obviously this patient has presented to late for this but thrombolysis is usually done with alteplase at 10% bolus, 90% infusion at a dose of 0.9 mg/kg. Presentation after the 4.5 hour window will be managed with aspirin.

The Bamford/Oxford Stroke Classification subtypes ischaemic stroke according to vascular territory of infarction. After initial management, stroke care involves the ethos of an MDT environment with rehabilitation. This is not a TIA which resolves within 24 hours of symptom onset, although in practice this is often difficult to distinguish due to interventional measures which are rapidly enacted.

114
Q

A 56 year old lady collapses whilst running for the bus. O/E there is a thrusting apex beat and an ejection systolic murmur. This is best heard on expiration and radiates to the carotids.

What is the most likely diagnosis?
A. Epilepsy
B. Vasovagal syncope
C. Cardiac arrhythmia
D. Stroke
E. Postural hypotension
F. TIA
G. Aortic stenosis
H. Anaemia
I. Hypoglycaemia
J. Pulmonary stenosis
K. Myxoedema coma
L. Carotid sinus sensitivity
A

G. Aortic stenosis

Aortic stenosis can present with chest pain, dyspnoea and syncope. It is characterised by a harsh ejection systolic murmur heard loudest at the right upper sternal edge at end expiration, which radiates up towards the carotids. The pulse pressure is narrow and there may be an associated slow-rising and plateau pulse. Doppler echo is vital for diagnosis and shows a pressure gradient across the narrowed valve orifice. 20% of cases are due to a congenital bicuspid valve. The most common cause of aortic stenosis in adults is calcification of normal trileaflet valves. Clinically stable patients may be considered for surgical repair or TAVR.

115
Q

A previously well 14 year old girl collapses after being in a crowd for 2 hours at a pop concert.

What is the most likely diagnosis?
A. Epilepsy
B. Vasovagal syncope
C. Cardiac arrhythmia
D. Stroke
E. Postural hypotension
F. TIA
G. Aortic stenosis
H. Anaemia
I. Hypoglycaemia
J. Pulmonary stenosis
K. Myxoedema coma
L. Carotid sinus sensitivity
A

B. Vasovagal syncope

This is vasovagal syncope which is also known as the ‘common faint’. It has many manifestations and many triggers such as emotional upset, unpleasant sights, fear and pain. These tend to recur and patients needs to be warned of this and that they are almost never life-threatening. The cause of the faint is a transient insufficiency of cerebral flow. LOC inevitably leads to loss of postural tone and the patient falling over.

116
Q

A 54 year old man collapses suddenly as he is walking across the living room. His daughter who witnessed the collapse says he dropped suddenly became very pale and started to twitch for a few seconds. After she woke him up (with some difficulty) he became flushed. O/E his nervous system is normal.

What is the most likely diagnosis?
A. Epilepsy
B. Vasovagal syncope
C. Cardiac arrhythmia
D. Stroke
E. Postural hypotension
F. TIA
G. Aortic stenosis
H. Anaemia
I. Hypoglycaemia
J. Pulmonary stenosis
K. Myxoedema coma
L. Carotid sinus sensitivity
A

C. Cardiac arrhythmia

Stokes-Adams attacks are episodes of transient LOC due to sudden decreased cardiac output. Pallor prior to the attack and facial flushing due to reactive hyperemia after the attack is characteristic of a Stokes-Adams attack. The underlying cause is a cardiac arrhythmia such as complete heart block.

5) Postural hypotension is a side effect of anti-hypertensives and is a common problem in the elderly. Diabetic autonomic neuropathy may also be a cause here. A good history should be enough to diagnose this.

117
Q

A 70 year old lady with a history of well-controlled diabetes, collapses when she gets out of bed in the morning. She regained consciousness and pulled the emergency cord in her bedroom to call for help. She is on medication for hypertension and her ECG is unremarkable.

What is the most likely diagnosis?
A. Epilepsy
B. Vasovagal syncope
C. Cardiac arrhythmia
D. Stroke
E. Postural hypotension
F. TIA
G. Aortic stenosis
H. Anaemia
I. Hypoglycaemia
J. Pulmonary stenosis
K. Myxoedema coma
L. Carotid sinus sensitivity
A

E. Postural hypotension

Postural hypotension is a side effect of anti-hypertensives and is a common problem in the elderly. Diabetic autonomic neuropathy may also be a cause here. A good history should be enough to diagnose this.

118
Q

A 71 year old man with long standing hypertension lives on his own. He is found by his son with marked right upper limb weakness and difficulties with speech. He is now incontinent of urine and has some personality changes.

What is the most likely diagnosis?
A. Peripheral neuropathy 
B. Myasthenia gravis 
C. Extradural haemorrhage
D. Multiple sclerosis
E. Depression 
F. Space-occupying lesion 
G. Subdural haemorrhage 
H. Arterial dissection
I. Stroke 
J. Migraine
K. Polio
A

I. Stroke

This is a left sided stroke which may have some frontal lobe involvement consistent with his personality changes. It is important is perform a CT head exclude a haemorrhagic aetiology and consider thrombolysis with tPA if within the 4.5 hour window and there are no contraindications. Thrombolysis is done with alteplase at 10% bolus, 90% infusion at a dose of 0.9 mg/kg. Presentation after the 4.5 hour window is managed with aspirin. The Bamford/Oxford Stroke Classification subtypes ischaemic stroke according to vascular territory of infarction. After initial management, stroke care involves the ethos of an MDT environment with rehabilitation.

119
Q

A 22-year-old female student recently lost her mother. Now she presents with a couple of weeks of increasing weakness in her arm and legs, but also while chewing or talking. Her vision seemed blurred recently.

What is the most likely diagnosis?
A. Peripheral neuropathy 
B. Myasthenia gravis 
C. Extradural haemorrhage
D. Multiple sclerosis
E. Depression 
F. Space-occupying lesion 
G. Subdural haemorrhage 
H. Arterial dissection
I. Stroke 
J. Migraine
K. Polio
A

B. Myasthenia gravis

Myasthenia gravis is an autoimmune condition with antibodies affecting the NMJ, mostly the nAChR at the post-synaptic muscle membrane. Although some have antibodies against MuSK, and there are other proteins involved. MG is characterised by muscle weakness which increases with exercise (fatigue, unlike Lambert-Eaton myasthenic syndrome). Commonly, presentations include diplopia and drooping eyelids like this patient, and there may also be SOB, proximal limb weakness, facial paresis and oropharyngeal weakness. MG is associated with thymic hyperplasia in 70% or thymoma in 10%, and these associations can also crop up in EMQs. There will be elevated serum AChR receptor antibody titres or MuSK antibodies. Electrophysiology will demonstrate a decremental response on repetitive nerve stimulation. Treatment includes anticholinesterases (pyridostigmine, and immunotherapy. Patients may also require a thymectomy. Some 15-20% may experience a myasthenic crisis (which needs mechanical ventilation). Do you know what the Tensilon test is and why edrophonium is given in this test?

120
Q

A mother of a 16-year old is concerned as he dropped a few things from his hands in the past two weeks. He says his arm went weak. He also complained of progressive headaches, associated with nausea and vomiting. He also had some visual disturbances.

What is the most likely diagnosis?
A. Peripheral neuropathy 
B. Myasthenia gravis 
C. Extradural haemorrhage
D. Multiple sclerosis
E. Depression 
F. Space-occupying lesion 
G. Subdural haemorrhage 
H. Arterial dissection
I. Stroke 
J. Migraine
K. Polio
A

F. Space-occupying lesion

Intracranial space-occupying lesions are mostly caused by primary and metastatic tumours. The progressive headache associated with N&V, visual disturbances and focal neurology indicate a slowly growing space-occupying mass. This could be a brain tumour, which may be primary (originates within the cranium) or secondary (from a metastatic tumour found elsewhere). The signs of raised intracranial pressure are seen here, and include headache, altered mental status, nausear and/or vomiting and gait abnormality. Tumours may be benign or malignant and range from meningiomas and medulloblastomas to craniopharyngiomas and astrocytic brain tumours. Further investigation is warranted in this case.

121
Q

A 56-year-old lady with T2DM presents with gradual onset of weakness of the left hand and right foot. On examination, you also notice reduced sensation in the lower legs. She tells you that she also suffers burning and shooting pain in her feet.

What is the most likely diagnosis?
A. Peripheral neuropathy 
B. Myasthenia gravis 
C. Extradural haemorrhage
D. Multiple sclerosis
E. Depression 
F. Space-occupying lesion 
G. Subdural haemorrhage 
H. Arterial dissection
I. Stroke 
J. Migraine
K. Polio
A

A. Peripheral neuropathy

This is by definition peripheral neuropathy. This is a case of diabetic neuropathy with sensory and motor deficits. This is a microvascular complication of DM and is characterised by peripheral nerve dysfunction. Pain is a common complaint such as the burning and shooting sensation this patient describes. Patient’s may also describe the pain as prickling or sticking. Examination should include peripheral pulses, reflexes and sensation to light touch with a 10g monofilament, vibration (128Hz tuning fork), pinprick and proprioception. The pain may be treated with medications like pregabalin (VGCC modulator) and duloxetine (serotonin-NA re-uptake inhibitor).

122
Q

A 41-year-old lady feels unsteady on her feet. She also has visual problems for the past 6 weeks. She has been previously well and has no other medical problems.

What is the most likely diagnosis?
A. Peripheral neuropathy 
B. Myasthenia gravis 
C. Extradural haemorrhage
D. Multiple sclerosis
E. Depression 
F. Space-occupying lesion 
G. Subdural haemorrhage 
H. Arterial dissection
I. Stroke 
J. Migraine
K. Polio
A

D. Multiple sclerosis

Multiple sclerosis is a demyelinating CNS condition which is characterised by 2 or more episodes of neurological dysfunction which are separated in both time and space. MS classically presents in white women aged 20-40 with temporary visual/sensory loss although any presentation can occur. MRI is a sensitive test but less specific than spinal MRI, however, spinal MRI is abnormal in fewer cases. Treatment aims at treating the attack, preventing future attacks and symptomatic treatment of problems like bladder dysfunction, pain and fatigue.

123
Q

A 87-year-old lady had a seizure at home. Her husband says she has been drowsy for the last couple of days. She is otherwise in good health but she has recently started antihpertensive drugs. On examination, her skin turgor is reduced.

What is the most likely diagnosis?
A. Tonic-clonic seizure 
B. Hyponatraemia 
C. Hypocalcaemia 
D. Atonic seizure 
E. Absence seizure
F. Encephalitis 
G. Jacksonian seizure 
H. Meningitis 
I. Hypercalcaemia 
J. Simple partial seizure
A

B. Hyponatraemia

Hyponatraemia is defined by serum sodium <135 with severe hyponatraemia defined as <120mmol/L. Thiazide diuretics are the most common cause of a hypovolaemic hyponatraemia. You get potassium loss with thiazides which contributes to an intracellular sodium shift and volume loss. This stimulates ADH secretion leading to water retention. Loops diuretics are actually less likely to cause hyponatraemia, possibly because they flush out the solute from the renal medulla reducing the medullary interstitial osmotic pressure.

There are many causes of hyponatraemia which can be classified by serum tonicity. Additionally, isotonic hyponatraemia is an artefact due to high levels of lipid or protein and is referred to as pseudohyponatraemia. When correcting this condition, it must not be done too rapidly to avoid a phenomenon known as central pontine myelinolysis, characterised by demyelination and irreversible axonal damage.

124
Q

A 66-year-old was getting increasing confused over the last couple of days. She was admitted after a seizure. She appears distressed and is pyrexial with mild meningism but no rash. A CT head scan shows changes in the left temporal lobe and cerebral oedema.

What is the most likely diagnosis?
A. Tonic-clonic seizure 
B. Hyponatraemia 
C. Hypocalcaemia 
D. Atonic seizure 
E. Absence seizure
F. Encephalitis 
G. Jacksonian seizure 
H. Meningitis 
I. Hypercalcaemia 
J. Simple partial seizure
A

F. Encephalitis

Encephalitis (brain parenchyma inflammation) is not to be confused with meningitis where the meninges is inflammed (although meningoencephalitis can exist). In encephalitis, there is an altered state of consciousness (this patient is confused) and may there also be focal neurology. There is also a fever (exceptions, however, are subacute sclerosing panencephalitis, VZV and HCV). Other typical features include headache and seizures. It is a medical emergency and people are the extremes of age are more at risk. The list of possible causes is massive. HSV encephalitis is characterised by temporal lobe changes. Hypodense lesions and mass effect can be seen in the temporal lobes on CT. An MRI is a better test for encephalitis although it is less available when compared to CT.

If you suspect this diagnosis, empiral IV acyclovir should be started before the results of any investigations are known. A large proportion are caused by HSV and empirical therapy is backed up by RCTs which show improved mortality. You can change the treatment if the cause is known, for instance, ganciclovir for CMV. You should look for a rash which may give a clue to the aetiology. For example, vesicular patterns are seen in VZV, HSV and enteroviruses, EBV causes a maculopapular pattern after ampicillin, Lyme disease pathognomically gives erythema migrans whereas erythema nodosum might make you think of TB. The patient may also have animal/insect bites and a careful and thorough history is important.

125
Q

A 42-year-old lady has a seizure the day after her thyroidectomy surgery. She is previous well and taking no medication. She has never had a seizure before.

What is the most likely diagnosis?
A. Tonic-clonic seizure 
B. Hyponatraemia 
C. Hypocalcaemia 
D. Atonic seizure 
E. Absence seizure
F. Encephalitis 
G. Jacksonian seizure 
H. Meningitis 
I. Hypercalcaemia 
J. Simple partial seizure
A

C. Hypocalcaemia

Hypocalcaemia can develop as a complication of thyroid surgery due to the loss of parathyroid glands which produce PTH. This is the cause of this lady’s seizure. Trousseau’s sign can be seen which is carpal spasm when a blood pressure cuff is used for several minutes. Carpopedal spasm that occurs with hypocalcaemia is a painful spasm and could also be the presenting sign. Chvostek’s sign is twitching of the perioral muscles in response to tapping over the facial nerve at the ear. If urgent replacement is necessary, calcium gluconate can be given IV. It is preferred over calcium chloride as it causes less tissue necrosis if it leaks out. It is worth noting that digoxin may be ineffective until serum calcium is restored to normal.

126
Q

A 21-year-old man is walking down the street to visit his friends while suddenly he falls to the ground unconscious. His body goes stiff and then he begins to jerk his arms. He becomes incontinent of urine.

What is the most likely diagnosis?
A. Tonic-clonic seizure 
B. Hyponatraemia 
C. Hypocalcaemia 
D. Atonic seizure 
E. Absence seizure
F. Encephalitis 
G. Jacksonian seizure 
H. Meningitis 
I. Hypercalcaemia 
J. Simple partial seizure
A

A. Tonic-clonic seizure

This is a tonic-clonic, generalised seizure. It is characterised by LOC and widespread motor tonic contractions followed by clonic jerking movements. There will characteristically be a suppressed level of arousal following the event. This may either reflect a primary generalised episode or a focal seizure with secondary generalisation. The main aim of acute treatment is to terminate the seizure and to protect the airway. Management always starts with basic life-support (like every acute emergency) and your ABCs. IV access needs to be established (bloods sent to the lab too and serum glucose measured to test for reversable causes of seizure activity – thiamine should also be given to the patient if there is any concern about deficiency and hypoglycaemia, for instance in alcohol abuse). The following are needed: ECG, pulse oximetry, ABG. IV lorazepam is the preferred initial therapy, though rectal diazepam can be used if there is no IV access. If BZDs fail to stop the seizure then phenytoin or fosphenytoin can be tried.

After the episode, MRI and EEG are essential in diagnosing an epilepsy syndrome. During the episode of generalised tonic-clonic activity, the EEG will show bilateral synchrony in the epileptiform activity. If this is a one-off seizure in which a provoking factor, such as electrolyte disturbance or hypoglycaemia, has been identified then there is no need for therapy for epilepsy. In unprovoked cases, this depends on history, examination, EEG and MRI. Treatment may not be needed the first time but after a second unprovoked instance, therapy is generally recommended.

127
Q

A 10-year-old girl is not doing well at school, her teacher says she doesn’t concentrate and shows no interest. Her parents also noticed the girl has moments (around 10 seconds) when she just stares blankly, blinking and then returns to normal. It happens up to several times an hour.

What is the most likely diagnosis?
A. Tonic-clonic seizure 
B. Hyponatraemia 
C. Hypocalcaemia 
D. Atonic seizure 
E. Absence seizure
F. Encephalitis 
G. Jacksonian seizure 
H. Meningitis 
I. Hypercalcaemia 
J. Simple partial seizure
A

E. Absence seizure

A typical absence seizure is characterised by behavioural arrest or staring, lasting 5-10 seconds, interrupting otherwise normal activity. It can be induced by hyperventilation. Absence seizures can also be atypical, where it is less clear when it begins and ends and is not usually precipitated by hyperventilation. The definitive test here is to do an EEG to determine the exact nature of the seizure. This will ensure appropriate treatment. Most are medically responsive and childhood absence epilepsy (CAE) tends to remit by adulthood. First line treatment is with ethosuximide, valproate or lamotrigine. Atypical seizures though, tend to be refractory to medical treatment and associated with mental retardation.

128
Q

A 72-year-old arteriopath is brought into the A&E collapsed. After admission you realise he is blind although he denies it.

What is the most likely diagnosis?
A. Normal pressure hydrocephalus
B. Pick's disease
C. Hypothyroidism
D. Occipital Stroke
E. Multiple Sclerosis
F. Lewy body dementia
G. Vascular dementia
H. Parkinson's disease
I. Azheimer's dementia
A

D. Occipital Stroke

This is cortical blindness, which is the loss of vision in a normal eye which results from damage to the visual area in the occipital cortex. The lack of insight that they have lost vision is a phenomenon known as Anton’s syndrome. They are ‘cortically blind’ but claim, often confabulate and adamantly despite obvious evidence, claim that they are capable of seeing. The pupillary reflex to light is intact as it does not involve the cortex and fundscopy, if done, would also be normal.

129
Q

A 74-year-old man feels unsteady on his feet. He is on no medication and has no PMH. O/E you notice his gait is slow with small steps. He has a resting tremor of his right hand, and you note the cogwheel rigidity of his upper limbs.

What is the most likely diagnosis?
A. Normal pressure hydrocephalus
B. Pick's disease
C. Hypothyroidism
D. Occipital Stroke
E. Multiple Sclerosis
F. Lewy body dementia
G. Vascular dementia
H. Parkinson's disease
I. Azheimer's dementia
A

H. Parkinson’s disease

Parkinson’s is characterised by a resting tremor, rigidity, bradykinesia and postural instability. This patient has difficulty walking, a resting tremor (4-6 Hz at rest which dissipates with the use of limbs, with generally asymmetrical onset) and limbs oppose movement. This last point demonstrates rigidity, which shows as resistance to passive movement about a joint. There is often also cogwheeling, especially if there is a superimposed tremor. The patient may have other signs like a mask like face due to the loss of spontaneous facial movement, hypophonia and micrographia, and may walk around in a shuffling gait with a stooped posture. The diagnosis is clinical. Treatment is symptomatic in an MDT setting. Medical therapy includes MAO-B inhibitors and DA agonists, for example rasagiline and carbidopa/levodopa. There are other medical therapies depending on the specific symptoms the patient presents with.

130
Q

A 75-year-old lady is brought to see her GP by her husband. He says that in the past couple of months she needs more and more assistance with everyday life as she is very forgetful, gets lost easily and cannot name objects at times. He is very upset as he says she is not the way she used to be. Examination and routine investigations are unremarkable although her MMSE is 19/30.

What is the most likely diagnosis?
A. Normal pressure hydrocephalus
B. Pick's disease
C. Hypothyroidism
D. Occipital Stroke
E. Multiple Sclerosis
F. Lewy body dementia
G. Vascular dementia
H. Parkinson's disease
I. Azheimer's dementia
A

I. Azheimer’s dementia

This is Alzheimer’s dementia which is a progressive irreversible disorder characterised by memory loss, loss of social function and dimished executive function. The MMSE is the most widely used screening test for cognitive function and a score <24 is widely accepted as abnormal. This disease runs a deteriorating course and lesions in the brain are characterised by neurofibrillary tangles, plaques of beta amyloid and neurone loss with cortical atrophy. Cholinesterase inhibitors can be used (donepezil, rivastigmine, galantamine). Depression is common in AD and antidepressants may also be indicated. Carer support is crucial and remains the mainstay of treatment with an MDT ethos involving for example, OTs to assess home safety.

131
Q

A 80-year-old gentleman became agitated recently, he has behavioural changes including sexual disinhibition. He is unable to take care of himself, being unable to plan or make judgments on even simplest matters. His MMSE is 25/30.

What is the most likely diagnosis?
A. Normal pressure hydrocephalus
B. Pick's disease
C. Hypothyroidism
D. Occipital Stroke
E. Multiple Sclerosis
F. Lewy body dementia
G. Vascular dementia
H. Parkinson's disease
I. Azheimer's dementia
A

B. Pick’s disease

Pick’s disease (not to be confused with Niemann-Pick disease) is one cause of frontotemporal degeneration. If you are not aware of this then this question would have been difficult. In any case, you should have realised that this is frontotemporal dementia. This type of dementia presents primarily with disruption in personality and social conduct, or as a primary language disorder. Essentially, the patient will present with components of the dysexecutive syndrome. Almost 50% will also display parkinsonism and a subset also have MND. Treatment is supportive combining medications ranging from BZDs to antidepressants depending on symptoms, with community services and carer guidance.

132
Q

A 72-year-old smoker, a known arteriopath is suffering from increasing behavioural problems and forgetfulness. His family is concerned as his state is deteriorating. This is confirmed by the decline of his MMSE from 25/30 six months ago to 18/30 today.

What is the most likely diagnosis?
A. Normal pressure hydrocephalus
B. Pick's disease
C. Hypothyroidism
D. Occipital Stroke
E. Multiple Sclerosis
F. Lewy body dementia
G. Vascular dementia
H. Parkinson's disease
I. Azheimer's dementia
A

G. Vascular dementia

Vascular dementia is a chronic and progressive dementia with loss of brain parenchyma mainly due to causes such as infarction and small vessel changes. It is classically assumed to be a stepwise progression in symptoms, although a gradual course can also be seen. This patient is an arteriopath. It is the second most common cause of dementia in older people and there is a large overlap with Alzheimer’s with many patients having a mixed form. Treatment is of limited use and the best course of action is to target vascular risk factors at as early a stage as possible.

133
Q

A 70 year old man is found by his wife to have difficulty speaking and comprehending spoken language. There is also an inability to raise his right arm. He was fine an hour ago when his sister spoke to him on the phone.

What is the most likely diagnosis?
A. Right anterior circulation stroke
B. Left anterior circulation stroke
C. Left posterior circulation stroke
D. Lacunar stroke
A

B. Left anterior circulation stroke

Anterior circulation strokes are commonly associated with aphasia and loss of strength in the face and upper and/or lower extremities. The inability to raise the right arm points to a left sided lesion. Remember that UMNs innervate contralateral LMNs, which innervate ipsilateral muscles.

134
Q

A 30 year old man with a history of alcohol abuse is found by the police lying on the side of the motorway. The man has severe ataxia on walking into A&E. On examination, he is clearly confused. There is horizontal gaze palsy with markedly impaired vestibulo-ocular reflexes.

What is the most likely diagnosis?
A. Alcohol withdrawal 
B. Viral encephalitis 
C. Miller-Fisher syndrome 
D. Wernicke's encephalopathy
A

D. Wernicke’s encephalopathy

Wernicke’s encephalopathy is caused by the acute deficiency of thiamine in a susceptible host. It is clinically underdiagnosed. The classic triad is of mental status changes, ophthalmoplegia and gait dysfunction, though this is present in only 10% of cases.

135
Q

An 8 year old boy with no significant PMH comes into clinic with worried parents who report frequent unusual episodes over the past few months. The parents tell you their child will suddenly stop activity for 10 to 20 seconds around 5 times a day and will be found staring with minimal eyelid flutter. During this time, he is unresponsive to voice.

What is the most likely diagnosis?
A. Daydreaming 
B. Frontal epilepsy 
C. Partial seizure 
D. Absence seizure
A

D. Absence seizure

This description is of a typical absence seizure which interrupts otherwise normal activity. This can be hyperventilation-induced. EEG is the definitive test and most cases are medically responsive.

136
Q

A 28 year old woman presents with severe weakness in the legs and distal arms. She began to have foot drop in her pre-school years which has progressed through her teens. She finds walking up stairs difficult and has been wheelchair bound since the age of 20. You notice the lower leg and foot are atrophied and there is pes cavus.

What is the most likely diagnosis?
A. Diabetic neuropathy
B. Charcot-Marie-Tooth disease
C. Hereditary spastic paraplegia
D. Motor neurone disease
A

B. Charcot-Marie-Tooth disease

This history here points to Charcot-Marie-Tooth disease. There is pes cavus and distal atrophy of the legs here which are characteristic features. The progressive history from childhood suggests this genetic condition as the cause.

137
Q

A 27 year old medical student complains of a throbbing pain on the left side of her head. This is accompanied by nausea and vomiting as well as photophobia. It lasts about 6 hours and beforehand, she claims she felt a tingling in her limbs.

What is the most likely diagnosis?
A. Subarachnoid haemorrhage 
B. Migraine 
C. Tension headache 
D. Meningitis
A

B. Migraine

This is a classic migraine history with an aura of a tingling sensation, which is pathognomic of migraine. The pain tends to be described as throbbing or pulsing and tends to be disabling, and can be associated with N&V and photophobia.

138
Q

A 54 year old man presents complaining of dizziness. He describes it as a sudden and severe spinning feeling which is precipitated by rolling over in bed onto his left side. Symptoms last under a minute and has occurred every night for a month.

What is the most likely diagnosis?
A. Benign positional paroxysmal vertigo
B. Meniere's disease
C. Vestibular neuritis
D. Labyrinthitis
A

A. Benign positional paroxysmal vertigo

The history of vertigo on rolling over in bed which lasts for a few seconds is suggestive here. Dix-Hallpike test would be diagnostic in this case and will demonstrate nystagmus and symptoms delayed by about 15s, peak in 20-30s and then decay with complete resolution of the episode of vertigo.

139
Q

A hypertensive old man with a history of atrial fibrillation is brought into A&E by his carer who is worried that he is getting confused. The carer tells you that he has fallen over frequently over the last few months and is unstable on his feet. There is right-sided pronator drift and some right sided weakness.

What is the most likely diagnosis?
A. Brain tumour
B. Subdural haematoma
C. Concussive syndrome
D. Benign intracranial hypertension
A

B. Subdural haematoma

There is a history of falls here and confusion. A subdural haematoma occurs due to extra-axial blood collection between the dura and arachnoid layers surrounding the brain. Surgical treatment will likely be necessary in this case with neurological deficit.

140
Q

An 18 month old child from rural India presents to the local clinic where you are based for your elective. He has weakness in his left leg. Approximately 2 weeks ago, he has had fever and diarrhoea which resolved without treatment. The boy’s mother only believes in herbal remedies. Examination reveals the left leg has decreased tone, power and reflexes.

What is the most likely diagnosis?
A. Guillan-Barre syndrome
B. Poliovirus infection 
C. Transverse myelitis 
D. Mycobacterium tuberculosis
A

B. Poliovirus infection

This is paralytic poliovirus infection. The mother here only believes in herbal remedies so this child has not been immunised. The gastrointestinal prodrome is common although non-specific. Examination findings here are consistent. While GBS may be preceded by gastroenteritis, the limb weakness is usually symmetrical and there may be autonomic dysfunction. There is no cure for poliovirus infection.

141
Q

A 22 year old female presents with recurrent slurred speech which gets worse as she continues to talk. Her friends think she is retarded. She also has difficulty swallowing which worsens as she continues to eat. There is also some double vision.

What is the most likely diagnosis?
A. Myasthenia gravis
B. Multiple sclerosis
C. Chronic subdural haematoma
D. Embolic stroke
A

A. Myasthenia gravis

MG is characterised by muscle weakness that increases with exercise and improves on rest. It is a chronic autoimmune disorder of the post-synaptic membrane at the NMJ in skeletal muscle.

142
Q

A 35 year old engineer with a 10 pack year history presents with cloudy vision and pain on movement of the left eye. He also has some problems discriminating colours. 3 weeks ago he had unilateral parathesias during exam time and also occasionally has some tingling in his arms and legs when he is stressed.

What is the most likely diagnosis?
A. Fibromyalgia
B. Migrain
C. Pituitary tumour
D. Multiple sclerosis
A

D. Multiple sclerosis

MS is a demyelinating CNS condition clinically defined by 2 episodes of neurological dysfunction which are separated in space and time. Smoking is a weak risk factor.

143
Q

A 54 year old postman has had lower back pain for 2 years now. Over the past year he has experienced bilateral leg pain and heavy legs when walking. This pain is relieved by sitting down. He has now assumed a stooped posture when walking which makes the pain better.

What is the most likely diagnosis?
A. Spinal stenosis
B. Lumbosacral disc herniation
C. Peripheral vascular disease
D. Spinal compression fracture
A

A. Spinal stenosis

This is neurogenic claudication due to spinal stenosis. This is relieved by sitting or bending over, which widens the size of the spinal canal. This is the reason for the stooped posture when walking, which is classically described as the ‘shopping cart sign’ where patients lean forwards like onto a shopping trolley to flex the spine.